Уравнения правила решения: Правила решения простых уравнений — Уравнения

Содержание

Общие сведения об уравнениях

Уравнения — одна из сложных тем для усвоения, но при этом они являются достаточно мощным инструментом для решения большинства задач.

С помощью уравнений описываются различные процессы, протекающие в природе. Уравнения широко применяются в других науках: в экономике, физике, биологии и химии.

В данном уроке мы попробуем понять суть простейших уравнений, научимся выражать неизвестные и решим несколько уравнений. По мере усвоения новых материалов, уравнения будут усложняться, поэтому понять основы очень важно.

Предварительные навыки

Что такое уравнение?

Уравнение — это равенство, содержащее в себе переменную, значение которой требуется найти. Это значение должно быть таким, чтобы при его подстановке в исходное уравнение получалось верное числовое равенство.

Например выражение 3 + 2 = 5 является равенством. При вычислении левой части получается верное числовое равенство 5 = 5.

А вот равенство 3 + x = 5 является уравнением, поскольку содержит в себе переменную

x, значение которой можно найти. Значение должно быть таким, чтобы при подстановке этого значения в исходное уравнение, получилось верное числовое равенство.

Другими словами, мы должны найти такое значение, при котором знак равенства оправдал бы свое местоположение — левая часть должна быть равна правой части.

Уравнение 3 + x = 5 является элементарным. Значение переменной x равно числу 2. При любом другом значении равенство соблюдáться не будет

Говорят, что число 2 является корнем или решением уравнения 3 + x = 5

Корень или решение уравнения — это значение переменной, при котором уравнение обращается в верное числовое равенство.

Корней может быть несколько или не быть совсем. Решить уравнение

означает найти его корни или доказать, что корней нет.

Переменную, входящую в уравнение, иначе называют неизвестным. Вы вправе называть как вам удобнее. Это синонимы.

Примечание. Словосочетание «решить уравнение» говорит самó за себя. Решить уравнение означает «уравнять» равенство — сделать его сбалансированным, чтобы левая часть равнялась правой части.


Выразить одно через другое

Изучение уравнений по традиции начинается с того, чтобы научиться выражать одно число, входящее в равенство, через ряд других. Давайте не будем нарушать эту традицию и поступим также.

Рассмотрим следующее выражение:

8 + 2

Данное выражение является суммой чисел 8 и 2. Значение данного выражения равно 10

8 + 2 = 10

Получили равенство. Теперь можно выразить любое число из этого равенства через другие числа, входящие в это же равенство. К примеру, выразим число 2.

Чтобы выразить число 2, нужно задать вопрос: «что нужно сделать с числами 10 и 8, чтобы получить число 2». Понятно, что для получения числа 2, нужно из числа 10 вычесть число 8.

Так и делаем. Записываем число 2 и через знак равенства говорим, что для получения этого числа 2 мы из числа 10 вычли число 8:

2 = 10 − 8

Мы выразили число 2 из равенства 8 + 2 = 10. Как видно из примера, ничего сложного в этом нет.

При решении уравнений, в частности при выражении одного числа через другие, знак равенства удобно заменять на слово «есть». Делать это нужно мысленно, а не в самом выражении.

Так, выражая число 2 из равенства 8 + 2 = 10 мы получили равенство 2 = 10 − 8. Данное равенство можно прочесть так:

2 есть 10 − 8

То есть знак =

заменен на слово «есть». Более того, равенство 2 = 10 − 8 можно перевести с математического языка на полноценный человеческий язык. Тогда его можно будет прочитать следующим образом:

Число 2 есть разность числа 10 и числа 8

или

Число 2 есть разница между числом 10 и числом 8.

Но мы ограничимся лишь заменой знака равенства на слово «есть», и то будем делать это не всегда. Элементарные выражения можно понимать и без перевода математического языка на язык человеческий.

Вернём получившееся равенство 2 = 10 − 8 в первоначальное состояние:

8 + 2 = 10

Выразим в этот раз число 8. Что нужно сделать с остальными числами, чтобы получить число 8? Верно, нужно из числа 10 вычесть число 2

8 = 10 − 2

Вернем получившееся равенство 8 = 10 − 2 в первоначальное состояние:

8 + 2 = 10

В этот раз выразим число 10. Но оказывается, что десятку выражать не нужно, поскольку она уже выражена. Достаточно поменять местами левую и правую часть, тогда получится то, что нам нужно:

10 = 8 + 2


Пример 2. Рассмотрим равенство 8 − 2 = 6

Выразим из этого равенства число 8. Чтобы выразить число 8 остальные два числа нужно сложить:

8 = 6 + 2

Вернем получившееся равенство 8 = 6 + 2 в первоначальное состояние:

8 − 2 = 6

Выразим из этого равенства число 2. Чтобы выразить число 2, нужно из 8 вычесть 6

2 = 8 − 6


Пример 3. Рассмотрим равенство 3 × 2 = 6

Выразим число 3. Чтобы выразить число 3, нужно 6 разделить 2

Вернем получившееся равенство  в первоначальное состояние:

3 × 2 = 6

Выразим из этого равенства число 2. Чтобы выразить число 2, нужно 6 разделить 3


Пример 4. Рассмотрим равенство 

Выразим из этого равенства число 15. Чтобы выразить число 15, нужно перемножить числа 3 и 5

15 = 3 × 5

Вернем получившееся равенство 15 = 3 × 5 в первоначальное состояние:

Выразим из этого равенства число 5. Чтобы выразить число 5, нужно 15 разделить 3


Правила нахождения неизвестных

Рассмотрим несколько правил нахождения неизвестных. Возможно, они вам знакомы, но не мешает повторить их ещё раз. В дальнейшем их можно будет забыть, поскольку мы научимся решать уравнения, не применяя эти правила.

Вернемся к первому примеру, который мы рассматривали в предыдущей теме, где в равенстве 8 + 2 = 10 требовалось выразить число 2.

В равенстве 8 + 2 = 10 числа 8 и 2 являются слагаемыми, а число 10 — суммой.

Чтобы выразить число 2, мы поступили следующим образом:

2 = 10 − 8

То есть из суммы 10 вычли слагаемое 8.

Теперь представим, что в равенстве 8 + 2 = 10 вместо числа 2 располагается переменная x

8 + x = 10

В этом случае равенство 8 + 2 = 10 превращается в уравнение 8 + = 10, а переменная x берет на себя роль так называемого неизвестного слагаемого

Наша задача найти это неизвестное слагаемое, то есть решить уравнение 8 + = 10. Для нахождения неизвестного слагаемого предусмотрено следующее правило:

Чтобы найти неизвестное слагаемое, нужно из суммы вычесть известное слагаемое.

Что мы в принципе и сделали, когда выражали двойку в равенстве 8 + 2 = 10. Чтобы выразить слагаемое 2, мы из суммы 10 вычли другое слагаемое 8

2 = 10 − 8

А сейчас, чтобы найти неизвестное слагаемое x, мы должны из суммы 10 вычесть известное слагаемое 8:

x = 10 − 8

Если вычислить правую часть получившегося равенства, то можно узнать чему равна переменная

x

x = 2

Мы решили уравнение. Значение переменной x равно 2. Для проверки значение переменной x отправляют в исходное уравнение 8 + = 10 и подставляют вместо x. Так желательно поступать с любым решённым уравнением, поскольку нельзя быть точно уверенным, что уравнение решено правильно:

В результате получается верное числовое равенство. Значит уравнение решено правильно.

Это же правило действовало бы в случае, если неизвестным слагаемым было бы первое число 8.

x + 2 = 10

В этом уравнении x — это неизвестное слагаемое, 2 — известное слагаемое, 10 — сумма. Чтобы найти неизвестное слагаемое x, нужно из суммы 10 вычесть известное слагаемое 2

x = 10 − 2

x = 8


Вернемся ко второму примеру из предыдущей темы, где в равенстве 8 − 2 = 6 требовалось выразить число 8.

В равенстве 8 − 2 = 6 число 8 это уменьшаемое, число 2 — вычитаемое, число 6 — разность

Чтобы выразить число 8, мы поступили следующим образом:

8 = 6 + 2

То есть сложили разность 6 и вычитаемое 2.

Теперь представим, что в равенстве 8 − 2 = 6 вместо числа 8 располагается переменная x

x − 2 = 6

В этом случае переменная x берет на себя роль так называемого неизвестного уменьшаемого

Для нахождения неизвестного уменьшаемого предусмотрено следующее правило:

Чтобы найти неизвестное уменьшаемое, нужно к разности прибавить вычитаемое.

Что мы и сделали, когда выражали число 8 в равенстве 8 − 2 = 6. Чтобы выразить уменьшаемое 8, мы к разности 6 прибавили вычитаемое 2.

А сейчас, чтобы найти неизвестное уменьшаемое x, мы должны к разности 6 прибавить вычитаемое 2

x = 6 + 2

Если вычислить правую часть, то можно узнать чему равна переменная x

x = 8


Теперь представим, что в равенстве 8 − 2 = 6 вместо числа 2 располагается переменная x

8 − x = 6

В этом случае переменная x берет на себя роль неизвестного вычитаемого

Для нахождения неизвестного вычитаемого предусмотрено следующее правило:

Чтобы найти неизвестное вычитаемое, нужно из уменьшаемого вычесть разность.

Что мы и сделали, когда выражали число 2 в равенстве 8 − 2 = 6. Чтобы выразить число 2, мы из уменьшаемого 8 вычли разность 6.

А сейчас, чтобы найти неизвестное вычитаемое x, нужно опять же из уменьшаемого 8 вычесть разность 6

x = 8 − 6

Вычисляем правую часть и находим значение x

x = 2


Вернемся к третьему примеру из предыдущей темы, где в равенстве 3 × 2 = 6 мы пробовали выразить число 3.

В равенстве 3 × 2 = 6 число 3 — это множимое, число 2 — множитель, число 6 — произведение

Чтобы выразить число 3 мы поступили следующим образом:

То есть разделили произведение 6 на множитель 2.

Теперь представим, что в равенстве 3 × 2 = 6 вместо числа 3 располагается переменная x

x × 2 = 6

В этом случае переменная x берет на себя роль неизвестного множимого.

Для нахождения неизвестного множимого предусмотрено следующее правило:

Чтобы найти неизвестное множимое, нужно произведение разделить на множитель.

Что мы и сделали, когда выражали число 3 из равенства 3 × 2 = 6. Произведение 6 мы разделили на множитель 2.

А сейчас для нахождения неизвестного множимого x, нужно произведение 6 разделить на множитель 2.

Вычисление правой части позволяет нам найти значение переменной x

x = 3

Это же правило применимо в случае, если переменная x располагается вместо множителя, а не множимого. Представим, что в равенстве 3 × 2 = 6 вместо числа 2 располагается переменная x.

В этом случае переменная x берет на себя роль неизвестного множителя. Для нахождения неизвестного множителя предусмотрено такое же, что и для нахождения неизвестного множимого, а именно деление произведения на известный множитель:

Чтобы найти неизвестный множитель, нужно произведение разделить на множимое.

Что мы и сделали, когда выражали число 2 из равенства 3 × 2 = 6. Тогда для получения числа 2 мы разделили произведение 6 на множимое 3.

А сейчас для нахождения неизвестного множителя x мы разделили произведение 6 на множимое 3.

Вычисление правой части равенства  позволяет узнать чему равно x

x = 2

Множимое и множитель вместе называют сомножителями. Поскольку правила нахождения множимого и множителя совпадают, мы можем сформулировать общее правило нахождения неизвестного сомножителя:

Чтобы найти неизвестный сомножитель, нужно произведение разделить на известный сомножитель.

Например, решим уравнение 9 × x = 18. Переменная x является неизвестным сомножителем. Чтобы найти этот неизвестный сомножитель, нужно произведение 18 разделить на известный сомножитель 9

Отсюда .

Решим уравнение × 3 = 27. Переменная x является неизвестным сомножителем. Чтобы найти этот неизвестный сомножитель, нужно произведение 27 разделить на известный сомножитель 3

Отсюда .


Вернемся к четвертому примеру из предыдущей темы, где в равенстве  требовалось выразить число 15. В этом равенстве число 15 — это делимое, число 5 — делитель, число 3 — частное.

Чтобы выразить число 15 мы поступили следующим образом:

15 = 3 × 5

То есть умножили частное 3 на делитель 5.

Теперь представим, что в равенстве  вместо числа 15 располагается переменная x

В этом случае переменная x берет на себя роль неизвестного делимого.

Для нахождения неизвестного делимого предусмотрено следующее правило:

Чтобы найти неизвестное делимое, нужно частное умножить на делитель.

Что мы и сделали, когда выражали число 15 из равенства . Чтобы выразить число 15, мы умножили частное 3 на делитель 5.

А сейчас, чтобы найти неизвестное делимое x, нужно частное 3 умножить на делитель 5

x = 3 × 5

Вычислим правую часть получившегося равенства. Так мы узнаем чему равна переменная x.

x = 15


Теперь представим, что в равенстве  вместо числа 5 располагается переменная x.

В этом случае переменная x берет на себя роль неизвестного делителя.

Для нахождения неизвестного делителя предусмотрено следующее правило:

Чтобы найти неизвестный делитель, нужно делимое разделить на частное.

Что мы и сделали, когда выражали число 5 из равенства . Чтобы выразить число 5, мы разделили делимое 15 на частное 3.

А сейчас, чтобы найти неизвестный делитель x, нужно делимое 15 разделить на частное 3

Вычислим правую часть получившегося равенства. Так мы узнаем чему равна переменная x.

x = 5

Итак, для нахождения неизвестных мы изучили следующие правила:

  • Чтобы найти неизвестное слагаемое, нужно из суммы вычесть известное слагаемое;
  • Чтобы найти неизвестное уменьшаемое, нужно к разности прибавить вычитаемое;
  • Чтобы найти неизвестное вычитаемое, нужно из уменьшаемого вычесть разность;
  • Чтобы найти неизвестное множимое, нужно произведение разделить на множитель;
  • Чтобы найти неизвестный множитель, нужно произведение разделить на множимое;
  • Чтобы найти неизвестное делимое, нужно частное умножить на делитель;
  • Чтобы найти неизвестный делитель, нужно делимое разделить на частное.

Компоненты

Компонентами мы будем называть числа и переменные, входящие в равенство

Так, компонентами сложения являются слагаемые и сумма


Компонентами вычитания являются уменьшаемое, вычитаемое и разность


Компонентами умножения являются множимое, множитель и произведение


Компонентами деления являются делимое, делитель и частное

В зависимости от того, с какими компонентами мы будем иметь дело, будут применяться соответствующие правила нахождения неизвестных. Эти правила мы изучили в предыдущей теме. При решении уравнений желательно знать эти правило наизусть.

Пример 1. Найти корень уравнения 45 + x = 60

45 — слагаемое, x — неизвестное слагаемое, 60 — сумма. Имеем дело с компонентами сложения. Вспоминаем, что для нахождения неизвестного слагаемого, нужно из суммы вычесть известное слагаемое:

x = 60 − 45

Вычислим правую часть, получим значение x равное 15

x = 15

Значит корень уравнения 45 + x = 60 равен 15.

Чаще всего неизвестное слагаемое необходимо привести к виду при котором его можно было бы выразить.

Пример 2. Решить уравнение 

Здесь в отличие от предыдущего примера, неизвестное слагаемое нельзя выразить сразу, поскольку оно содержит коэффициент 2. Наша задача привести это уравнение к виду при котором можно было бы выразить x

В данном примере мы имеем дело с компонентами сложения — слагаемыми и суммой. 2x — это первое слагаемое, 4 — второе слагаемое, 8 — сумма.

При этом слагаемое 2x содержит переменную x. После нахождения значения переменной x слагаемое 2x примет другой вид. Поэтому слагаемое 2x можно полностью принять за неизвестное слагаемое:

Теперь применяем правило нахождения неизвестного слагаемого. Вычитаем из суммы известное слагаемое:

Вычислим правую часть получившегося уравнения:

Мы получили новое уравнение . Теперь мы имеем дело с компонентами умножения: множимым, множителем и произведением. 2 — множимое, — множитель, 4 — произведение

При этом переменная x является не просто множителем, а неизвестным множителем

Чтобы найти этот неизвестный множитель, нужно произведение разделить на множимое:

Вычислим правую часть, получим значение переменной x

Для проверки найденный корень отправим в исходное уравнение  и подставим вместо x

Получили верное числовое равенство. Значит уравнение решено правильно.


Пример 3. Решить уравнение 3+ 9+ 16= 56

Cразу выразить неизвестное x нельзя. Сначала нужно привести данное уравнение к виду при котором его можно было бы выразить.

Приведем подобные слагаемые в левой части данного уравнения:

Имеем дело с компонентами умножения. 28 — множимое, — множитель, 56 — произведение. При этом x является неизвестным множителем. Чтобы найти неизвестный множитель, нужно произведение разделить на множимое:

Отсюда x равен 2


Равносильные уравнения

В предыдущем примере при решении уравнения 3x + 9x + 16x = 56, мы привели подобные слагаемые в левой части уравнения. В результате получили новое уравнение 28x = 56. Старое уравнение 3x + 9x + 16x = 56 и получившееся новое уравнение 28x = 56 называют равносильными уравнениями, поскольку их корни совпадают.

Уравнения называют равносильными, если их корни совпадают.

Проверим это. Для уравнения 3+ 9+ 16= 56 мы нашли корень равный 2. Подставим этот корень сначала в уравнение 3+ 9+ 16= 56, а затем в уравнение 28= 56, которое получилось в результате приведения подобных слагаемых в левой части предыдущего уравнения. Мы должны получить верные числовые равенства

Согласно порядку действий, в первую очередь выполняется умножение:

Подставим корень 2 во второе уравнение 28= 56

Видим, что у обоих уравнений корни совпадают. Значит уравнения 3+ 9+ 16= 56 и 28= 56 действительно являются равносильными.

Для решения уравнения 3+ 9+ 16= 56 мы воспользовались одним из тождественных преобразований — приведением подобных слагаемых. Правильное тождественное преобразование уравнения позволило нам получить равносильное уравнение 28= 56, которое проще решать.

Из тождественных преобразований на данный момент мы умеем только сокращать дроби, приводить подобные слагаемые, выносить общий множитель за скобки, а также раскрывать скобки. Существуют и другие преобразования, которые следует знать. Но для общего представления о тождественных преобразованиях уравнений, изученных нами тем вполне хватает.


Рассмотрим некоторые преобразования, которые позволяют получить равносильное уравнение

Если к обеим частям уравнения прибавить одно и то же число, то получится уравнение равносильное данному.

и аналогично:

Если из обеих частей уравнения вычесть одно и то же число, то получится уравнение равносильное данному.

Другими словами, корень уравнения не изменится, если к обеим частям данного уравнения прибавить (или вычесть из обеих частей) одно и то же число.

Пример 1. Решить уравнение

Вычтем из обеих частей уравнения число 10

Приведем подобные слагаемые в обеих частях:

Получили уравнение 5= 10. Имеем дело с компонентами умножения. Чтобы найти неизвестный сомножитель x, нужно произведение 10 разделить на известный сомножитель 5.

Отсюда .

Вернемся к исходному уравнению  и подставим вместо x найденное значение 2

Получили верное числовое равенство. Значит уравнение решено правильно.

Решая уравнение мы вычли из обеих частей уравнения число 10. В результате получили равносильное уравнение . Корень этого уравнения, как и уравнения  так же равен 2


Пример 2. Решить уравнение 4(+ 3) = 16

Раскроем скобки в левой части равенства:

Вычтем из обеих частей уравнения число 12

Приведем подобные слагаемые в обеих частях уравнения:

В левой части останется 4x, а в правой части число 4

 

 

Получили уравнение 4= 4. Имеем дело с компонентами умножения. Чтобы найти неизвестный сомножитель x, нужно произведение 4 разделить на известный сомножитель 4

Отсюда 

Вернемся к исходному уравнению 4(+ 3) = 16 и подставим вместо x найденное значение 1

 

Получили верное числовое равенство. Значит уравнение решено правильно.

Решая уравнение 4(+ 3) = 16 мы вычли из обеих частей уравнения число 12. В результате получили равносильное уравнение 4= 4. Корень этого уравнения, как и уравнения 4(+ 3) = 16 так же равен 1


Пример 3. Решить уравнение

Раскроем скобки в левой части равенства:

Прибавим к обеим частям уравнения число 8

Приведем подобные слагаемые в обеих частях уравнения:

В левой части останется 2x, а в правой части число 9

В получившемся уравнении 2= 9 выразим неизвестное слагаемое x

 

Отсюда 

Вернемся к исходному уравнению  и подставим вместо x найденное значение 4,5

Получили верное числовое равенство. Значит уравнение решено правильно.

Решая уравнение  мы прибавили к обеим частям уравнения число 8. В результате получили равносильное уравнение . Корень этого уравнения, как и уравнения  так же равен 4,5


Следующее правило, которое позволяет получить равносильное уравнение, выглядит следующим образом

Если в уравнении перенести слагаемое из одной части в другую, изменив его знак, то получится уравнение равносильное данному.

То есть корень уравнения не изменится, если мы перенесем слагаемое из одной части уравнения в другую, изменив его знак. Это свойство является одним из важных и одним из часто используемых при решении уравнений.

Рассмотрим следующее уравнение:

Корень данного уравнения равен 2. Подставим вместо x этот корень и проверим получается ли верное числовое равенство

Получается верное равенство. Значит число 2 действительно является корнем уравнения .

Теперь попробуем поэкспериментировать со слагаемыми этого уравнения, перенося их из одной части в другую, изменяя знаки.

Например, слагаемое 3x располагается в левой части равенства. Перенесём его в правую часть, изменив знак на противоположный:

Получилось уравнение 12 = 9x − 3x. Приведем подобные слагаемые в правой части данного уравнения:

Имеем дело с компонентами умножения. Переменная x является неизвестным сомножителем. Найдём этот известный сомножитель:

Отсюда = 2. Как видим, корень уравнения не изменился. Значит уравнения 12 + 3x = 9x и 12 = 9x − 3x являются равносильными.

На самом деле данное преобразование является упрощенным методом предыдущего преобразования, где к обеим частям уравнения прибавлялось (или вычиталось) одно и то же число.

Мы сказали, что в уравнении 12 + 3x = 9x слагаемое 3x было перенесено в правую часть, изменив знак. В реальности же происходило следующее: из обеих частей уравнения вычли слагаемое 3x

Затем в левой части были приведены подобные слагаемые и получено уравнение 12 = 9x − 3x. Затем опять были приведены подобные слагаемые, но уже в правой части, и получено уравнение 12 = 6x.

Но так называемый «перенос» более удобен для подобных уравнений, поэтому он и получил такое широкое распространение. Решая уравнения, мы часто будем пользоваться именно этим преобразованием.

Равносильными также являются уравнения 12 + 3= 9x и 3x − 9= −12. В этот раз в уравнении 12 + 3= 9x слагаемое 12 было перенесено в правую часть, а слагаемое 9x в левую. Не следует забывать, что знаки этих слагаемых были изменены во время переноса


Следующее правило, которое позволяет получить равносильное уравнение, выглядит следующим образом:

Если обе части уравнения умножить или разделить на одно и то же число, не равное нулю, то получится уравнение равносильное данному.

Другими словами, корни уравнения не изменятся, если обе его части умножить или разделить на одно и то же число. Это действие часто применяется тогда, когда нужно решить уравнение содержащее дробные выражения.

Сначала рассмотрим примеры, в которых обе части уравнения будут умножаться на одно и то же число.

Пример 1. Решить уравнение 

При решении уравнений, содержащих дробные выражения, сначала  принято упростить это уравнение.

В данном случае мы имеем дело именно с таким уравнением. В целях упрощения данного уравнения обе его части можно умножить на 8:

Мы помним, что для умножения дроби на число, нужно числитель данной дроби умножить на это число. У нас имеются две дроби и каждая из них умножается на число 8. Наша задача умножить числители дробей на это число 8

Теперь происходит самое интересное. В числителях и знаменателях обеих дробей содержится множитель 8, который можно сократить на 8. Это позволит нам избавиться от дробного выражения:

В результате останется простейшее уравнение

Ну и нетрудно догадаться, что корень этого уравнения равен 4

Вернемся к исходному уравнению   и подставим вместо x найденное значение 4

Получается верное числовое равенство. Значит уравнение решено правильно.

При решении данного уравнения мы умножили обе его части на 8. В результате получили уравнение . Корень этого уравнения, как и уравнения  равен 4. Значит эти уравнения равносильны.

Множитель на который умножаются обе части уравнения принято записывать перед частью уравнения, а не после неё. Так, решая уравнение , мы умножили обе части на множитель 8 и получили следующую запись:

От этого корень уравнения не изменился, но если бы мы сделали это находясь в школе, то нам сделали бы замечание, поскольку в алгебре множитель принято записывать перед тем выражением, с которым он перемножается. Поэтому умножение обеих частей уравнения  на множитель 8 желательно переписать следующим образом:


Пример 2. Решить уравнение 

Умнóжим обе части уравнения на 15

В левой части множители 15 можно сократить на 15, а в правой части множители 15 и 5 можно сократить на 5

Перепишем то, что у нас осталось:

Раскроем скобки в правой части уравнения:

Перенесем слагаемое x из левой части уравнения в правую часть, изменив знак. А слагаемое 15 из правой части уравнения перенесем в левую часть, опять же изменив знак:

Приведем подобные слагаемые в обеих частях, получим

Имеем дело с компонентами умножения. Переменная x является неизвестным сомножителем. Найдём этот известный сомножитель:

Отсюда 

Вернемся к исходному уравнению   и подставим вместо найденное значение 5

Получается верное числовое равенство. Значит уравнение решено правильно. При решении данного уравнения мы умножили обе го части на 15. Далее выполняя тождественные преобразования, мы получили уравнение 10 = 2x. Корень этого уравнения, как и уравнения  равен 5. Значит эти уравнения равносильны.


Пример 3. Решить уравнение 

Умнóжим обе части уравнения на 3

В левой части можно сократить две тройки, а правая часть будет равна 18

Останется простейшее уравнение . Имеем дело с компонентами умножения. Переменная x является неизвестным сомножителем. Найдём этот известный сомножитель:

Отсюда 

Вернемся к исходному уравнению   и подставим вместо найденное значение 9

Получается верное числовое равенство. Значит уравнение решено правильно.


Пример 4. Решить уравнение 

Умнóжим обе части уравнения на 6

В левой части уравнения раскроем скобки. В правой части множитель 6 можно поднять в числитель:

Сократим в обеих частях уравнениях то, что можно сократить:

Перепишем то, что у нас осталось:

Раскроем скобки в обеих частях уравнения:

Воспользуемся переносом слагаемых. Слагаемые, содержащие неизвестное x, сгруппируем в левой части уравнения, а слагаемые свободные от неизвестных — в правой:

Приведем подобные слагаемые в обеих частях:

Теперь найдем значение переменной x. Для этого разделим произведение 28 на известный сомножитель 7

Отсюда = 4.

Вернемся к исходному уравнению  и подставим вместо x найденное значение 4

Получилось верное числовое равенство. Значит уравнение решено правильно.


Пример 5. Решить уравнение 

Раскроем скобки в обеих частях уравнения там, где это можно:

Умнóжим обе части уравнения на 15

Раскроем скобки в обеих частях уравнения:

Сократим в обеих частях уравнения, то что можно сократить:

Перепишем то, что у нас осталось:

Раскроем скобки там, где это можно:

Воспользуемся переносом слагаемых. Слагаемые, содержащие неизвестное, сгруппируем в левой части уравнения, а слагаемые, свободные от неизвестных — в правой. Не забываем, что во время переноса, слагаемые меняют свои знаки на противоположные:

Приведем подобные слагаемые в обеих частях уравнения:

Найдём значение x

В получившемся ответе можно выделить целую часть:

Вернемся к исходному уравнению и подставим вместо x найденное значение 

Получается довольно громоздкое выражение. Воспользуемся переменными. Левую часть равенства занесем в переменную A, а правую часть равенства в переменную B

Наша задача состоит в том, чтобы убедиться равна ли левая часть правой. Другими словами, доказать равенство A = B

Найдем значение выражения, находящегося в переменной А.

Значение переменной А равно . Теперь найдем значение переменной B. То есть значение правой части нашего равенства. Если и оно равно , то уравнение будет решено верно

Видим, что значение переменной B, как и значение переменной A равно . Это значит, что левая часть равна правой части. Отсюда делаем вывод, что уравнение решено правильно.

Теперь попробуем не умножать обе части уравнения на одно и то же число, а делить.

Рассмотрим уравнение 30+ 14+ 14 = 70− 40+ 42. Решим его обычным методом: слагаемые, содержащие неизвестные, сгруппируем в левой части уравнения, а слагаемые, свободные от неизвестных — в правой. Далее выполняя известные тождественные преобразования, найдем значение x

Подставим найденное значение 2 вместо x в исходное уравнение:

Теперь попробуем разделить все слагаемые уравнения 30+ 14+ 14 = 70− 40+ 42 на какое-нибудь число. Замечаем, что все слагаемые этого уравнения имеют общий множитель 2. На него и разделим каждое слагаемое:

Выполним сокращение в каждом слагаемом:

Перепишем то, что у нас осталось:

Решим это уравнение, пользуясь известными тождественными преобразованиями:

Получили корень 2. Значит уравнения 15+ 7+ 7 = 35x − 20+ 21 и 30+ 14+ 14 = 70− 40+ 42 равносильны.

Деление обеих частей уравнения на одно и то же число позволяет освобождать неизвестное от коэффициента. В предыдущем примере когда мы получили уравнение 7= 14, нам потребовалось разделить произведение 14 на известный сомножитель 7. Но если бы мы в левой части освободили неизвестное от коэффициента 7, корень нашелся бы сразу. Для этого достаточно было разделить обе части на 7

Этим методом мы тоже будем пользоваться часто.


Умножение на минус единицу

Если обе части уравнения умножить на минус единицу, то получится уравнение равносильное данному.

Это правило следует из того, что от умножения (или деления) обеих частей уравнения на одно и то же число, корень данного уравнения не меняется. А значит корень не поменяется если обе его части умножить на −1.

Данное правило позволяет поменять знаки всех компонентов, входящих в уравнение. Для чего это нужно? Опять же, чтобы получить равносильное уравнение, которое проще решать.

Рассмотрим уравнение . Чему равен корень этого уравнения?

Прибавим к обеим частям уравнения число 5

Приведем подобные слагаемые:

А теперь вспомним про коэффициент буквенного выражения. Что же представляет собой левая часть уравнения . Это есть произведение минус единицы и переменной x

То есть минус, стоящий перед переменной x, относится не к самой переменной x, а к единице, которую мы не видим, поскольку коэффициент 1 принято не записывать. Это означает, что уравнение  на самом деле выглядит следующим образом:

Имеем дело с компонентами умножения. Чтобы найти х, нужно произведение −5 разделить на известный сомножитель −1.

или разделить обе части уравнения на −1, что еще проще

Итак, корень уравнения  равен 5. Для проверки подставим его в исходное уравнение. Не забываем, что в исходном уравнении минус стоящий перед переменной x относится к невидимой единице

Получилось верное числовое равенство. Значит уравнение решено верно.

Теперь попробуем умножить обе части уравнения  на минус единицу:

После раскрытия скобок в левой части образуется выражение , а правая часть будет равна 10

Корень этого уравнения, как и уравнения  равен 5

Значит уравнения  и  равносильны.


Пример 2. Решить уравнение 

В данном уравнении все компоненты являются отрицательными. С положительными компонентами работать удобнее, чем с отрицательными, поэтому поменяем знаки всех компонентов, входящих в уравнение . Для этого умнóжим обе части данного уравнения на −1.

Понятно, что от умножения на −1 любое число поменяет свой знак на противоположный. Поэтому саму процедуру умножения на −1 и раскрытие скобок подробно не расписывают, а сразу записывают компоненты уравнения с противоположными знаками.

Так, умножение уравнения  на −1 можно записать подробно следующим образом:

либо можно просто поменять знаки всех компонентов:

Получится то же самое, но разница будет в том, что мы сэкономим себе время.

Итак, умножив обе части уравнения  на −1, мы получили уравнение . Решим данное уравнение. Из обеих частей вычтем число 4 и разделим обе части на 3

Когда корень найден, переменную обычно записывают в левой части, а её значение в правой, что мы и сделали.


Пример 3. Решить уравнение 

Умнóжим обе части уравнения на −1. Тогда все компоненты поменяют свои знаки на противоположные:

Из обеих частей получившегося уравнения вычтем 2x и приведем подобные слагаемые:

Прибавим к обеим частям уравнения единицу и приведем подобные слагаемые: 


Приравнивание к нулю

Недавно мы узнали, что если в уравнении перенести слагаемое из одной части в другую, изменив его знак, то получится уравнение равносильное данному.

А что будет если перенести из одной части в другую не одно слагаемое, а все слагаемые? Верно, в той части откуда забрали все слагаемые останется ноль. Иными словами, не останется ничего.

В качестве примера рассмотрим уравнение . Решим данное уравнение, как обычно — слагаемые, содержащие неизвестные сгруппируем в одной части, а числовые слагаемые, свободные от неизвестных оставим в другой. Далее выполняя известные тождественные преобразования, найдем значение переменной x

Теперь попробуем решить это же уравнение, приравняв все его компоненты к нулю. Для этого перенесем все слагаемые из правой части в левую, изменив знаки:

Приведем подобные слагаемые в левой части:

Прибавим к обеим частям 77, и разделим обе части на 7


Альтернатива правилам нахождения неизвестных

Очевидно, что зная о тождественных преобразованиях уравнений, можно не заучивать наизусть правила нахождения неизвестных.

К примеру, для нахождения неизвестного в уравнении  мы произведение 10 делили на известный сомножитель 2

Но если в уравнении  обе части разделить на 2 корень найдется сразу. В левой части уравнения в числителе множитель 2 и в знаменателе множитель 2 сократятся на 2. А правая часть будет  равна 5

Уравнения вида  мы решали выражая неизвестное слагаемое:

Но можно воспользоваться тождественными преобразованиями, которые мы сегодня изучили. В уравнении слагаемое 4 можно перенести в правую часть, изменив знак:

Далее разделить обе части на 2

В левой части уравнения сократятся две двойки. Правая часть будет равна 2. Отсюда .

Либо можно было из обеих частей уравнения вычесть 4. Тогда получилось бы следующее:

В случае с уравнениями вида  удобнее делить произведение на известный сомножитель. Сравним оба решения:

Первое решение намного короче и аккуратнее. Второе решение можно значительно укоротить, если выполнить деление в уме.

Тем не менее, необходимо знать оба метода, и только затем использовать тот, который больше нравится.


Когда корней несколько

Уравнение может иметь несколько корней. Например уравнение x(x + 9) = 0 имеет два корня: 0 и −9.

В уравнении x(x + 9) = 0 нужно было найти такое значение при котором левая часть была бы равна нулю. В левой части этого уравнения содержатся выражения x и (x + 9), которые являются сомножителями. Из законов умножения мы знаем, что произведение равно нулю, если хотя бы один из сомножителей равен нулю (или первый сомножитель или второй).

То есть в уравнении x(x + 9) = 0 равенство будет достигаться, если x будет равен нулю или (x + 9) будет равно нулю.

x = 0 или x + 9 = 0

Приравняв к нулю оба этих выражения, мы сможем найти корни уравнения x(x + 9) = 0. Первый корень, как видно из примера, нашелся сразу. Для нахождения второго корня нужно решить элементарное уравнение + 9 = 0. Несложно догадаться, что корень этого уравнения равен −9. Проверка показывает, что корень верный:

−9 + 9 = 0


Пример 2. Решить уравнение

Данное уравнение имеет два корня: 1 и 2. Левая часть уравнения является произведение выражений (x − 1) и (x − 2). А произведение равно нулю, если хотя бы один из сомножителей равен нулю (или сомножитель (x − 1) или сомножитель (x − 2)).

Найдем такое x при котором выражения (x − 1) или (x − 2) обращаются в нули:

Подставляем по-очереди найденные значения в исходное уравнение  и убеждаемся, что при этих значениях левая часть равняется нулю:


Когда корней бесконечно много

Уравнение может иметь бесконечно много корней. То есть подставив в такое уравнение любое число, мы получим верное числовое равенство.

Пример 1. Решить уравнение 

Корнем данного уравнения является любое число. Если раскрыть скобки в левой части уравнения и привести подобные слагаемые, то получится равенство 14 = 14. Это равенство будет получаться при любом x


Пример 2. Решить уравнение 

Корнем данного уравнения является любое число. Если раскрыть скобки в левой части уравнения, то получится равенство 10x + 12 = 10x + 12. Это равенство будет получаться при любом x


Когда корней нет

Случается и так, что уравнение вовсе не имеет решений, то есть не имеет корней. Например уравнение не имеет корней, поскольку при любом значении x, левая часть уравнения не будет равна правой части. Например, пусть . Тогда уравнение примет следующий вид

Пусть


Пример 2. Решить уравнение 

Раскроем скобки в левой части равенства:

Приведем подобные слагаемые:

Видим, что левая часть не равна правой части. И так будет при любом значении y. Например, пусть y = 3.


Буквенные уравнения

Уравнение может содержать не только числа с переменными, но и буквы.

Например, формула нахождения скорости является буквенным уравнением:

Данное уравнение описывает скорость движения тела при равноускоренном движении.

Полезным навыком является умение выразить любой компонент, входящий в буквенное уравнение. Например, чтобы из уравнения  определить расстояние, нужно выразить переменную s.

Умнóжим обе части уравнения  на t

В правой части переменные t сократим на t и перепишем то, что у нас осталось:

В получившемся уравнении левую и правую часть поменяем местами:

У нас получилась формула нахождения расстояния, которую мы изучали ранее.

Попробуем из уравнения  определить время. Для этого нужно выразить переменную t.

Умнóжим обе части уравнения на t

В правой части переменные t сократим на t и перепишем то, что у нас осталось:

В получившемся уравнении v × t = s обе части разделим на v

В левой части переменные v сократим на v и перепишем то, что у нас осталось:

У нас получилась формула определения времени, которую мы изучали ранее.

Предположим, что скорость поезда равна 50 км/ч

v = 50 км/ч

А расстояние равно 100 км

s = 100 км

Тогда буквенное уравнение примет следующий вид

Из этого уравнения можно найти время. Для этого нужно суметь выразить переменную t. Можно воспользоваться правилом нахождения неизвестного делителя, разделив делимое на частное и таким образом определить значение переменной t

либо можно воспользоваться тождественными преобразованиями. Сначала умножить обе части уравнения на t

Затем разделить обе части на 50


Пример 2. Дано буквенное уравнение . Выразите из данного уравнения x

Вычтем из обеих частей уравнения a

Разделим обе части уравнения на b

Теперь, если нам попадется уравнение вида a + bx = c, то у нас будет готовое решение. Достаточно будет подставить в него нужные значения. Те значения, которые будут подставляться вместо букв a, b, c принято называть параметрами. А уравнения вида a + bx = c называют уравнением с параметрами. В зависимости от параметров, корень будет меняться.

Решим уравнение 2 + 4x = 10. Оно похоже на буквенное уравнение a + bx = c.  Вместо того, чтобы выполнять тождественные преобразования, мы можем воспользоваться готовым решением. Сравним оба решения:

Видим, что второе решение намного проще и короче.

Для готового решения необходимо сделать небольшое замечание. Параметр b не должен быть равным нулю (b ≠ 0), поскольку деление на ноль на допускается.


Пример 3. Дано буквенное уравнение . Выразите из данного уравнения x

Раскроем скобки в обеих частях уравнения

Воспользуемся переносом слагаемых. Параметры, содержащие переменную x, сгруппируем в левой части уравнения, а параметры свободные от этой переменной — в правой.

В левой части вынесем за скобки множитель x

Разделим обе части на выражение a − b

В левой части числитель и знаменатель можно сократить на a − b. Так окончательно выразится переменная x

Теперь, если нам попадется уравнение вида a(x − c) = b(x + d), то у нас будет готовое решение. Достаточно будет подставить в него нужные значения.

Допустим нам дано уравнение 4(x − 3) = 2(+ 4). Оно похоже на уравнение a(x − c) = b(x + d). Решим его двумя способами: при помощи тождественных преобразований и при помощи готового решения:

Для удобства вытащим из уравнения 4(x − 3) = 2(+ 4) значения параметров a, b, c, d. Это позволит нам не ошибиться при подстановке:

Как и в прошлом примере знаменатель здесь не должен быть равным нулю (a − b ≠ 0). Если нам встретится уравнение вида a(x − c) = b(x + d) в котором параметры a и b будут одинаковыми, мы сможем не решая его сказать, что у данного уравнения корней нет, поскольку разность одинаковых чисел равна нулю.

Например, уравнение 2(x − 3) = 2(x + 4) является уравнением вида a(x − c) = b(x + d). В уравнении 2(x − 3) = 2(x + 4) параметры a и b одинаковые. Если мы начнём его решать, то придем к тому, что левая часть не будет равна правой части:


Пример 4. Дано буквенное уравнение . Выразите из данного уравнения x

Приведем левую часть уравнения к общему знаменателю:

Умнóжим обе части на a

В левой части x вынесем за скобки

Разделим обе части на выражение (1 − a)


Линейные уравнения с одним неизвестным

Рассмотренные в данном уроке уравнения называют линейными уравнениями первой степени с одним неизвестным.

Если уравнение дано в первой степени, не содержит деления на неизвестное, а также не содержит корней из неизвестного, то его можно назвать линейным. Мы еще не изучали степени и корни, поэтому чтобы не усложнять себе жизнь, слово «линейный» будем понимать как «простой».

Большинство уравнений, решенных в данном уроке, в конечном итоге сводились к простейшему уравнению, в котором нужно было произведение разделить на известный сомножитель. Таковым к примеру является уравнение 2(x + 3) = 16. Давайте решим его.

Раскроем скобки в левой части уравнения, получим 2+ 6 = 16. Перенесем слагаемое 6 в правую часть, изменив знак. Тогда получим 2= 16 − 6. Вычислим правую часть, получим 2= 10. Чтобы найти x, разделим произведение 10 на известный сомножитель 2. Отсюда x = 5.

Уравнение 2(x + 3) = 16 является линейным. Оно свелось к уравнению 2= 10, для нахождения корня которого потребовалось разделить произведение на известный сомножитель. Такое простейшее уравнение называют линейным уравнением первой степени с одним неизвестным в каноническом виде. Слово «канонический» является синонимом слов «простейший» или «нормальный».

Линейное уравнение первой степени с одним неизвестным в каноническом виде называют уравнение вида ax = b.

Полученное нами уравнение 2= 10 является линейным уравнением первой степени с одним неизвестным в каноническом виде. У этого уравнения первая степень, одно неизвестное, оно не содержит деления на неизвестное и не содержит корней из неизвестного, и представлено оно в каноническом виде, то есть в простейшем виде при котором легко можно определить значение x. Вместо параметров a и b в нашем уравнении содержатся числа 2 и 10. Но подобное уравнение может содержать и другие числа: положительные, отрицательные или равные нулю.

Если в линейном уравнении a = 0 и b = 0, то уравнение имеет бесконечно много корней. Действительно, если a равно нулю и b равно нулю, то линейное уравнение ax b примет вид 0= 0. При любом значении x левая часть будет равна правой части.

Если в линейном уравнении a = 0 и b ≠ 0, то уравнение корней не имеет. Действительно, если a равно нулю и b равно какому-нибудь числу, не равному нулю, скажем числу 5, то уравнение ax = b примет вид 0= 5. Левая часть будет равна нулю, а правая часть пяти. А ноль не равен пяти.

Если в линейном уравнении a ≠ 0, и b равно любому числу, то уравнение имеет один корень. Он определяется делением параметра b на параметр a

Действительно, если a равно какому-нибудь числу, не равному нулю, скажем числу 3, и b равно какому-нибудь числу, скажем числу 6, то уравнение  примет вид .
Отсюда .

Существует и другая форма записи линейного уравнения первой степени с одним неизвестным. Выглядит она следующим образом: ax − b = 0. Это то же самое уравнение, что и ax = b, но параметр b перенесен в левую часть с противоположным знаком. Такие уравнение мы тоже решали в данном уроке. Например, уравнение 7− 77 = 0. Уравнение вида ax − b = 0 называют линейным уравнением первой степени с одним неизвестным в общем виде.

В будущем после изучения рациональных выражений, мы рассмотрим такие понятия, как посторонние корни и потеря корней. А пока рассмотренного в данном уроке будет достаточным.

Задания для самостоятельного решения

Задание 1. Используя метод переноса слагаемого, решите следующее уравнение:

Задание 2. Используя метод прибавления (или вычитания) числа к обеим частям, решите следующее уравнение:

Задание 3. Решите уравнение:

Задание 4. Решите уравнение:

Задание 5. Решите уравнение:

Задание 6. Решите уравнение:

Задание 7. Решите уравнение:

Задание 8. Решите уравнение:

Задание 9. Решите уравнение:

Задание 10. Решите уравнение:

Задание 11. Решите уравнение:

Задание 12. Решите уравнение:

Задание 13. Решите уравнение:

Задание 14. Решите уравнение:

Задание 15. Решите уравнение:

Задание 16. Решите уравнение:

Задание 17. Решите уравнение:

Задание 18. Решите уравнение:

Задание 19. Решите уравнение:

Задание 20. Решите уравнение:

Задание 21. Решите уравнение:

Задание 22. Решите уравнение:

Задание 23. Решите уравнение:

Задание 24. Решите уравнение:

Задание 25. Решите уравнение:

Задание 26. Решите уравнение:

Задание 27. Решите уравнение:

Задание 28. Решите уравнение:

Задание 29. Решите уравнение:

Задание 30. Решите уравнение:

Задание 31. Решите уравнение:

Задание 32. В следующем буквенном уравнении выразите переменную x:

Задание 33. В следующем буквенном уравнении выразите переменную x:

Задание 34. В следующем буквенном уравнении выразите переменную x:

Задание 35. В следующем буквенном уравнении выразите переменную x:

Задание 36. В следующем буквенном уравнении выразите переменную y:

Задание 37. В следующем буквенном уравнении выразите переменную z:


Понравился урок?
Вступай в нашу новую группу Вконтакте и начни получать уведомления о новых уроках

Возникло желание поддержать проект?
Используй кнопку ниже

Навигация по записям

Карта сайта

  • О центре
    • Новости Института
    • Наши достижения
    • Наша команда
    • Фотоальбом
    • Вакансии
    • Контакты офиса
    • Магазин в Москве («Абрис»)
  • «Школа 2000…» учителям
    • Технология ДМ
    • Курс «Математика 1-9»
    • Курс «Мир деятельности»
    • Каллиграфия цифр
    • Международный конкурс «Учу учиться»
      • Положение о конкурсе
      • Список конкурсных работ
      • Правила оформления
    • Взаимодействие с родителями
    • Библиотека
  • «Школа 2000…» родителям
    • Важное о программе
    • Детская Академия Петерсон
    • Преимущества программы
    • Детские сады и школы
    • Шпаргалки для родителей
    • Основные риски
    • Курс «Мир деятельности»
      • О надпредметном курсе и авторах
      • Программа надпредметного курса для НШ и ОШ
      • Письмо об использовании надпредметного курса «Мир деятельности» в основной школе
      • Комплект для учителя
      • Комплект для ученика
      • Дополнительные материалы
      • Консультации к урокам
      • Отзывы о курсе
      • Комплекты «Мир деятельности»
    • Родительское собрание
    • В кабинете психолога
    • Библиотека для родителей
    • Поучительные притчи
    • Афоризмы об образовании
    • «Решебник» к учебникам
    • Родителям дошкольников
    • Мы в соцсетях
  • Учебники и методическая литература
    • Новинки
    • Концепция программы
    • Дошкольная подготовка
    • «Мир деятельности»
    • Начальная школа
    • Основная школа
    • Электронные приложения
    • Сценарии уроков на CD
  • Курсы повышения квалификации
    • Вебинары
    • Выездные курсы
    • Для работников дошкольного образования
    • Учителям начальной школы
    • Учителям основной школы
    • Курсы для заведующих, ППС, методистов кафедр математического образования
    • Стажировки
    • Сводное расписание курсов
    • Регистрация на курсы On-line
    • Дистанционное обучение
    • Отзывы о курсах
  • Дистанционное обучение
  • Нормативные документы, письма и программы
    • Правоустанавливающие документы
    • Актуальные документы
    • ООП для школы
    • Примерные рабочие программы по математике
    • Курс «Мир деятельности»
    • Государственный стандарт
    • Рекомендованные учебники
    • О функционировании Центра
    • О присуждении премий
    • Благодарственные письма
    • ООП для детского сада
    • Дошкольное образование
  • «Мир деятельности»
  • Прошедшие мероприятия
    • Конференции
    • Курсы
    • Семинары
    • Вебинары
    • Отзывы о курсах
  • Текущие проекты
    • Экспериментальная площадка
  • Вопросы и ответы
  • Библиотека
    • Библиотека для учителей
    • Из опыта работы
    • Библиотека для родителей
  • Контакты

Решение уравнений

— Добрый день, мои дорогие друзья! Сегодня мы с вами будем учиться решать уравнения.

А что же такое уравнение?

Помните, в первом классе вы решали примеры, в которых были пропущены числа?

Для того, чтобы вставить число в таких примерах, надо было вспомнить состав чисел в пределах 10.

А теперь вместо окошечек вы будете записывать буквы латинского алфавита:

Эти буквы сейчас используют в английском, немецком, французском и многих других языках. Вот посмотрите, как будут выглядеть наши примеры, в которых вместо окошек появились латинские буквы:

И называются они теперь — уравнения. Вы спросите, почему их так назвали? Да потому, что вместо буквы надо подставить такое число, чтобы уравнять левую и правую части выражения.

Уравнение — это математическое равенство, которое содержит неизвестное число. Но каждая ли запись, в которое есть неизвестное число является уравнением?

Давайте среди приведённых записей найдём уравнение:

Первая запись — это равенство, но в нём нет букв латинского алфавита. Значит это не уравнение.

Вторая запись. Конечно, и эта запись не будет являться уравнением, ведь это неравенство.

Следующая запись. Это равенство и оно содержит латинскую букву. Значит, эту запись мы назовём уравнением.

И ещё одна запись. Конечно это не уравнение, ведь эта запись не является равенством.

Итак, среди приведённых записей уравнением является третья запись. Давайте попробуем его решить.

А что значит «решить уравнение»?

Решить уравнение — значит, найти такое числовое значение неизвестного, при котором равенство будет верным.

В математике говорят так: «решить уравнение — значит найти корень уравнения». Корень уравнения — это то число, которое можно подставить вместо буквы.

Те уравнения с окошечками, которые были в первом классе, решать было легко. Выучил состав чисел в пределах 10, и подставляй нужное число. А вот если уравнение с двузначными числами, или с трёхзначными? Тут знание состава однозначных чисел нам не поможет.

Как же найти для решения нашего уравнения такое число, при котором получится верное равенство, т.е. найти корень уравнения?

Конечно, для того, чтобы найти верный способ решения уравнений, необходимо помнить правила:

·                   Чтобы найти неизвестное слагаемое, надо из суммы вычесть известное слагаемое.

·                   Чтобы найти неизвестное вычитаемое, надо из уменьшаемого вычесть разность.

·                   Чтобы найти неизвестное уменьшаемое, надо к разности прибавить вычитаемое.

Сейчас попробуем решить наше уравнение 45 + x = 68.

В этом уравнении неизвестным является слагаемое. Чтобы найти неизвестное слагаемое, надо из суммы вычесть известное слагаемое.

Поэтому получаем:

Давайте выполним проверку, уточним, верно ли мы нашли неизвестное число.

Вновь записываем наше уравнение, но вместо буквы икс пишем число 23:

Слева и в справа получили одно и тоже число значит, уравнение решено верно.

Как я уже говорила, для того:

·                   Чтобы найти неизвестное слагаемое, надо из суммы вычесть известное слагаемое.

·                   Чтобы найти неизвестное вычитаемое, надо из уменьшаемого вычесть разность.

·                   Чтобы найти неизвестное уменьшаемое, надо к разности прибавить вычитаемое.

То есть, надо знать три правила. Но я вам предлагаю ещё один способ выбора действия при решении уравнений.

Представьте себе яблоко. Сейчас оно целое. А если мы его разрежем и отодвинем одну часть, у нас останется вторая часть. Отодвигая, мы выполняли действие вычитание. Значит, чтобы найти часть, надо выполнить действие вычитание. А теперь давайте вернём назад нашу часть. У нас опять получилось целое яблоко. Чтобы получить целое яблоко, мы сложили части. А теперь представим себе это схематически:

Теперь все наши уравнения мы будем соотносить с полученными схемами.

Вот, например, такое уравнение:

К какой схеме оно подходит? Т.к. в нём стоит знак плюс оно подходит к первой схеме. Теперь мы видим, что в данном уравнении нам надо найти часть. Значит, мы из целого, суммы,  вычитаем известную часть — слагаемое. Получаем:

Давайте проверим. Записываем наше уравнение, только вместо буквы запишем полученное число, получаем:

Ответ: а = 25.

В нашем уравнении было неизвестно слагаемое. Чтобы найти неизвестное слагаемое, надо из суммы вычесть известное слагаемое. Это мы и сделали.

Решим ещё одно уравнение:

Посмотрим, к какой схеме оно подходит. В нём стоит знак минус. Значит ко второй. Теперь мы видим, что в данном уравнении нам надо найти целое. Вспомним, что целое находится сложением — складываем части. Получим:

Выполним проверку:

Уравнение решено верно, то есть найден корень уравнения. Он равен 46.

В этом уравнении нам были известны вычитаемое и разность. Неизвестно уменьшаемое. Чтобы найти неизвестное уменьшаемое, надо к разности прибавить вычитаемое. Что мы и сделали.

Ну и давайте решим ещё одно уравнение:

В этом уравнении, как и в предыдущем также выполняется вычитание. Но здесь известно уменьшаемое и разность, а неизвестно вычитаемое. Опять подставляем уравнение к схеме. Нам надо найти вычитаемое, т.е. часть. А как его найти? Часть всегда находится вычитанием. Надо из целого, т.е. уменьшаемого вычесть часть, т.е. разность.

Проверяем:

Получили верное равенство. Значит, уравнение решено верно, и число 50 является корнем уравнения. Нам надо было найти неизвестное вычитаемое, и мы из уменьшаемого вычитали разность.

Уравнения мы решили, а теперь давайте повторим то, что вы сегодня узнали на уроке.

При решении уравнений необходимо знать правила:

·                   Чтобы найти неизвестное слагаемое, надо из суммы вычесть известное слагаемое.

·                   Чтобы найти неизвестное вычитаемое, надо из уменьшаемого вычесть разность.

·                   Чтобы найти неизвестное уменьшаемое, надо к разности прибавить вычитаемое.

Для того чтобы безошибочно решать уравнения запомните наши схемы. Они всегда подскажут вам, какой способ решения уравнений нужно выбрать. Если надо найти целое, мы выполняем действие сложение. А если часть, то вычитание. А теперь обратите внимание на алгоритм решения уравнений:

1) Определить неизвестный компонент (что нужно найти — слагаемое, уменьшаемое или вычитаемое).

2) Применить правило нахождения неизвестного:

·                   Чтобы найти неизвестное слагаемое, надо из суммы вычесть известное слагаемое.

·                   Чтобы найти неизвестное вычитаемое, надо из уменьшаемого вычесть разность.

·                   Чтобы найти неизвестное уменьшаемое, надо к разности прибавить вычитаемое.

Выполнить действие и получить корень уравнения.

3) Выполнить проверку.

Постарайтесь запомнить все эти правила и тогда вы без труда сможете решать уравнения, т.е. находить их корни.

А я прощаюсь с вами и желаю вам в этом успехов при решении уравнений.

Примеры решения квадратных и биквадратных уравнений

Пример 4. Решить квадратное уравнение x2 + 12x + 36 = 0.

Решение.
Вычислим дискриминант квадратного трехчлена. У нас a = 1, b = 12, c = 36.

Так как b = 12 — четное число, то вычислим дискриминант D1 :

D1 = (b/2)2 — ac = 62 — 1*36 = 0, следовательно, уравнение имеет единственный корень x = (-b/2)/a = (-6)/1 = -6.

Это уравнение можно решить и без вычисления дискриминанта, преобразовав квадратный трехчлен по формуле сокращенного умножения:
x2 + 12x + 36 = 0 (x+6)2 = 0 x = -6.

Ответ: -6.

Пример 5. Решить квадратное уравнение 4x2 -28x + 49 = 0.

Решение.
Вычислим дискриминант квадратного трехчлена. У нас a = 4, b = -28, c = 49.

Так как b = -28 — четное число, то вычислим дискриминант D1 :

D1 = (b/2)2 — ac = (-14)2 — 4*49 = 196-196 = 0, следовательно, уравнение имеет единственный корень x = (-b/2)/a = 14/4 = 7/2.

Это уравнение также можно решить без вычисления дискриминанта, преобразовав квадратный трехчлен по формуле сокращенного умножения:

4x2 -28x + 49 = 0 (2x-7)2 = 0 2x = 7 x = 7/2.

Ответ: 7/2.

Пример 6. Решить уравнение .

Решение.
Приведем к общему знаменателю левую часть уравнения:

Умножив обе части уравнения на -4, получим x2 + 3x = 0. Это неполное квадратное уравнение решим способом разложения на множители:
x2 + 3x = 0 x(x+3) = 0

x = 0, x = 0,
x — 3 = 0 x = 3.

Ответ: 0, 3.

Пример 7. Решить уравнение .

Решение.
Приведем к общему знаменателю левую часть и правую части уравнения:

Получим 6x2 + 3x = 20x-10 6x2 + 3x — 20x + 10 = 0 6x2 — 17x + 10 = 0.

Вычислим дискриминант квадратного трехчлена: a = 6, b = -17, c = 10,
D = b2 — 4ac = (-17)2 — 4*6*10 = 289 — 240 = 49 > 0, следовательно, уравнение имеет два действительных корня.

Ответ: 5/6, 2.

Пример 8. Решить уравнение .

Решение.
Вычислим дискриминант квадратного трехчлена. У нас a = 1, b = 2√2, c = 1.

Так как b = 2√2, то есть b делится на 2 (b/2 = √2), вычислим дискриминант D1:

D1 = (b/2)2 — ac = (√2)2 — 1*1 = 1 > 0. Cледовательно, уравнение имеет два действительных корня.

Ответ: -√2-1, -√2+1.

Пример 9. Решить уравнение .

Решение.
Умножим левую и правую части уравнения на 6:

Вычислим дискриминант полученного квадратного трехчлена. У нас a = 3, b = -6, c = 2.

Так как b = -6, то есть b делится на 2 (b/2=3), вычислим дискриминант D1:

D1 = (b/2)2 — ac = 32 — 3*2 = 3 > 0. Cледовательно, уравнение имеет два действительных корня.

Ответ: (3-√3)/3, (3+√3)/3.

Пример 10. Решить уравнение x4 — 17x2 + 16 = 0.

Решение.
Исходное уравнение является биквадратным. Сделав замену переменной t = x2 => x4 = t2, перейдем к эквивалентному исходному квадратному уравнению:

x4 — 17x2 + 16 = 0 => t2 — 17t + 16 = 0.

Вычислим дискриминант квадратного трехчлена: a = 1, b = -17, c = 16,

D = b2 — 4ac = (-17)2 — 4*1*16 = 289-64 = 225 > 0, следовательно, уравнение имеет два действительных корня.

По найденным значениям t, решая уравнения x2 = t, найдем корни исходного биквадратного уравнения:

Таким образом, исходное уравнение имеет 4 действительных корня.

Ответ: ±1, ±4.

Пример 11. Решить уравнение 9x4 + 32x2 — 16 = 0.

Решение.
Исходное уравнение является биквадратным. Сделав замену переменной t = x2 => x4 = t2, перейдем к эквивалентному исходному квадратному уравнению:

9x4 + 32x2 — 16 = 0 => 9t2 + 32t — 16 = 0

Вычислим дискриминант квадратного трехчлена. У нас a = 9, b = 32, c = -16.

Так как b = 32, то есть b делится на 2 (b/2=16), вычислим дискриминант D1:

D1 = (b/2)2 — ac = 162 — 9*(-16) = 400 >0. Cледовательно, уравнение имеет два действительных корня.

По найденным значениям t, решая уравнения x2 = t, найдем корни исходного биквадратного уравнения:

Первое уравнение x2 = -4 корней не имеет, а второе, а значит, и исходное, имеет два действительных корня x= ±2/3.

Ответ: ±2/3.

Пример 12. Решить уравнение x4 + 3x2 — 10 = 0.

Решение.
Исходное уравнение является биквадратным. Сделав замену переменной t = x2 => x4 = t2, перейдем к эквивалентному исходному квадратному уравнению:

x4 + 3x2 — 10 = 0 => t2 + 3t — 10 = 0

Вычислим дискриминант полученного квадратного трехчлена: a = 1, b = 3, c = -10,

D = b2 — 4ac = 32 — 4*1*(-10) = 9+40 = 49 > 0, следовательно, уравнение имеет два действительных корня.

По найденным значениям t, решая уравнения x2 = t, найдем корни исходного биквадратного уравнения:

Первое уравнение x2 = -5 корней не имеет, а второе, а значит, и исходное, имеет два действительных корня x = ±√2.

Ответ: ±√2.

Решение линейных уравнений с примерами. Уравнения онлайн Примеры уравнений 5

Уравнение с одним неизвестным, которое после раскрытия скобок и приведения подобных членов принимает вид

aх + b = 0 , где a и b произвольные числа, называется линейным уравнением с одним неизвестным. Cегодня разберёмся, как эти линейные уравнения решать.

Например, все уравнения:

2х + 3= 7 – 0,5х; 0,3х = 0; x/2 + 3 = 1/2 (х – 2) — линейные.

Значение неизвестного, обращающее уравнение в верное равенство называется решением или корнем уравнения .

Например, если в уравнении 3х + 7 = 13 вместо неизвестного х подставить число 2 , то получим верное равенство 3· 2 +7 = 13. Значит, значение х = 2 есть решение или корень уравнения.

А значение х = 3 не обращает уравнение 3х + 7 = 13 в верное равенство, так как 3· 2 +7 ≠ 13. Значит, значение х = 3 не является решением или корнем уравнения.

Решение любых линейных уравнений сводится к решению уравнений вида

aх + b = 0.

Перенесем свободный член из левой части уравнения в правую, изменив при этом знак перед b на противоположный, получим

Если a ≠ 0, то х = ‒ b/a .

Пример 1. Решите уравнение 3х + 2 =11.

Перенесем 2 из левой части уравнения в правую, изменив при этом знак перед 2 на противоположный, получим
3х = 11 – 2.

Выполним вычитание, тогда
3х = 9.

Чтобы найти х надо разделить произведение на известный множитель, то есть
х = 9: 3.

Значит, значение х = 3 является решением или корнем уравнения.

Ответ: х = 3 .

Если а = 0 и b = 0 , то получим уравнение 0х = 0. Это уравнение имеет бесконечно много решений, так как при умножении любого числа на 0 мы получаем 0,но b тоже равно 0. Решением этого уравнения является любое число.

Пример 2. Решите уравнение 5(х – 3) + 2 = 3 (х – 4) + 2х ‒ 1.

Раскроем скобки:
5х – 15 + 2 = 3х – 12 + 2х ‒ 1.


5х – 3х ‒ 2х = – 12 ‒ 1 + 15 ‒ 2.

Приведем подобные члены:
0х = 0.

Ответ: х — любое число .

Если а = 0 и b ≠ 0 , то получим уравнение 0х = — b. Это уравнение решений не имеет, так как при умножении любого числа на 0 мы получаем 0, но b ≠ 0 .

Пример 3. Решите уравнение х + 8 = х + 5.

Сгруппируем в левой части члены, содержащие неизвестные, а в правой ‒ свободные члены:
х – х = 5 ‒ 8.

Приведем подобные члены:
0х = ‒ 3.

Ответ: нет решений.

На рисунке 1 изображена схема решения линейного уравнения

Составим общую схему решения уравнений с одной переменной. Рассмотрим решение примера 4.

Пример 4. Пусть надо решить уравнение

1) Умножим все члены уравнения на наименьшее общее кратное знаменателей, равное 12.

2) После сокращения получим
4 (х – 4) + 3·2 (х + 1) ‒ 12 = 6·5 (х – 3) + 24х – 2 (11х + 43)

3) Чтобы отделить члены, содержащие неизвестные и свободные члены, раскроем скобки:
4х – 16 + 6х + 6 – 12 = 30х – 90 + 24х – 22х – 86 .

4) Сгруппируем в одной части члены, содержащие неизвестные, а в другой – свободные члены:
4х + 6х – 30х – 24х + 22х = ‒ 90 – 86 + 16 – 6 + 12.

5) Приведем подобные члены:
‒ 22х = ‒ 154.

6) Разделим на – 22 , Получим
х = 7.

Как видим, корень уравнения равен семи.

Вообще такие уравнения можно решать по следующей схеме :

а) привести уравнение к целому виду;

б) раскрыть скобки;

в) сгруппировать члены, содержащие неизвестное, в одной части уравнения, а свободные члены ‒ в другой;

г) привести подобные члены;

д) решить уравнение вида aх = b,которое получили после приведения подобных членов.

Однако эта схема не обязательна для всякого уравнения. При решении многих более простых уравнений приходится начинать не с первого, а со второго (Пример. 2 ), третьего (Пример. 1, 3 ) и даже с пятого этапа, как в примере 5.

Пример 5. Решите уравнение 2х = 1/4.

Находим неизвестное х = 1/4: 2,
х = 1/8
.

Рассмотрим решение некоторых линейных уравнений, встречающихся на основном государственном экзамене.

Пример 6. Решите уравнение 2 (х + 3) = 5 – 6х.

2х + 6 = 5 – 6х

2х + 6х = 5 – 6

Ответ: ‒ 0, 125

Пример 7. Решите уравнение – 6 (5 – 3х) = 8х – 7.

– 30 + 18х = 8х – 7

18х – 8х = – 7 +30

Ответ: 2,3

Пример 8. Решите уравнение

3(3х – 4) = 4 · 7х + 24

9х – 12 = 28х + 24

9х – 28х = 24 + 12

Пример 9. Найдите f(6), если f (x + 2) = 3 7-х

Решение

Так как надо найти f(6), а нам известно f (x + 2),
то х + 2 = 6.

Решаем линейное уравнение х + 2 = 6,
получаем х = 6 – 2, х = 4.

Если х = 4, тогда
f(6) = 3 7-4 = 3 3 = 27

Ответ: 27.

Если у Вас остались вопросы, есть желание разобраться с решением уравнений более основательно, записывайтесь на мои уроки в РАСПИСАНИИ . Буду рада Вам помочь!

Также TutorOnline советует посмотреть новый видеоурок от нашего репетитора Ольги Александровны, который поможет разобраться как с линейными уравнениями, так и с другими.

сайт, при полном или частичном копировании материала ссылка на первоисточник обязательна.

Решаем дробно-рациональное уравнение 5/х = 100. Данное уравнение можно решить двумя способами. Давайте рассмотрим каждый из них.

План решения уравнения 5/x = 100

  • найдем область допустимых значений для заданного уравнения;
  • первый способ решения уравнения рассмотрев его как на пропорцию;
  • второй способ решения уравнения, находя неизвестный делитель.

Находим неизвестный член пропорции

Сначала найдем ОДЗ уравнения. В левой части уравнения присутствует знак дроби и он равносилен знаку деления. Известно, что на ноль делить нельзя. Значит из ОДЗ мы должны исключить значения обращающие знаменатель в ноль.

ОДЗ: x принадлежит R \ {0}.

Теперь посмотрим на наше уравнение как на пропорцию.

Основное свойство пропорции.

Произведение крайних членов пропорции равно произведению ее средних членов.

Для пропорции a: b = c: d или a/b = c/d основное свойство записывается так: a · d = b · c.

Применим его и получим линейное уравнение:

100 * x = 5 * 1;

Разделим на 100 обе части уравнения, тем самым избавимся от коэффициента перед переменной х:

Находим неизвестный делитель

Посмотрим на уравнение как на частное. Где делимое равно 5, делитель x, а результат деления — частное равно 100.

Вспомним правило как найти неизвестный делитель — нужно делимое разделить на частное.

Найденный корень принадлежит ОДЗ уравнения.

Проверим найденное решение уравнения. Для этого подставим найденные корень в исходное уравнение и произведем вычисления:

Решение найдено верно.

Одним из самых важных навыков при поступлении в 5 класс является умение решать простейшие уравнения. Так как 5 класс ещё не так далек от начальной школы, то и видов уравнений, которые может решать ученик не так уж и много. Мы познакомим Вас со всеми основными видами уравнений, которые необходимо уметь решать, если Вы хотите поступить в физико-математическую школу .

1 тип: «луковичные»
Это уравнения, которые почти со вероятностью встретятся Вам при поступлении в любую школу или кружок 5 класса как отдельное задание. Их легко отличить от других: в них переменная присутствует только 1 раз. Например, или .
Решаются они очень просто: необходимо просто «добраться» до неизвестной, постепенно «снимая» всё лишнее, что окружает её — как будто почистить луковицу — отсюда и такое название. Для решения достаточно помнить несколько правил из второго класса. Перечислим их все:

Сложение

  1. слагаемое1 + слагаемое2 = сумма
  2. слагаемое1 = сумма — слагаемое2
  3. слагаемое2 = сумма — слагаемое1

Вычитание

  1. уменьшаемое — вычитаемое = разность
  2. уменьшаемое = вычитаемое + разность
  3. вычитаемое = уменьшаемое — разность

Умножение

  1. множитель1 * множитель2 = произведение
  2. множитель1 = произведение: множитель2
  3. множитель2 = произведение: множитель1

Деление

  1. делимое: делитель = частное
  2. делимое = делитель * частное
  3. делитель = делимое: частное

Разберём на примере, как применять данные правила.

Заметим, что мы делим на и получаем . В этой ситуации мы знаем делитель и частное. Чтобы найти делимое, нужно делитель умножить на частное:

Мы стали немного ближе к самому . Теперь мы видим, что к прибавляется и получается . Значит, чтобы найти одно из слагаемых, нужно из суммы вычесть известное слагаемое:

И ещё один «слой» снят с неизвестной! Теперь мы видим ситуацию с известным значением произведения () и одним известным множителем ().

Теперь ситуация «уменьшаемое — вычитаемое = разность»

И последний шаг — известное произведение () и один из множителей ()

2 тип: уравнения со скобками
Уравнения данного типа чаще всего встречаются в задачах — именно к ним сводится 90% всех задач для поступления в 5 класс . В отличие от «луковичных уравнений» переменная здесь может встретиться несколько раз, поэтому решить её методами из предыдущего пункта невозможно. Типичные уравнения: или
Основная трудность — это правильно раскрыть скобки. После того, как удалось это верно сделать, следует привести подобные слагаемые (числа к числам, переменные к переменным), а после этого мы получаем самое простое «луковичное уравнение» , которое умеем решать. Но обо всём по-порядку.

Раскрытие скобок . Мы приведём несколько правил, которыми следует пользоваться в данном случае. Но, как показывает практика, верно раскрывать скобки ученик начинает только после 70-80 прорешанных задач. Основное правило таково: любой множитель, стоящий за скобками необходимо умножить на каждое слагаемое внутри скобок. А минус, стоящий перед скобкой, меняет знак всех выражений, что стоят внутри. Итак, основные правила раскрытия:



Приведение подобных . Здесь всё гораздо легче: Вам необходимо путём переноса слагаемых через знак равенства добиться того, чтобы с одной стороны стояли только слагаемые с неизвестной, а с другой — только числа. Основное правило таково: каждое слагаемое, переносимое через , меняет свой знак — если оно было с ,то станет с , и наоборот. После успешного переноса необходимо сосчитать итоговое количество неизвестных, итоговое число стоящее с другой стороны равенства, нежели переменные, и решить простое «луковичное уравнение» .

Приложение

Решение любого типа уравнений онлайн на сайт для закрепления изученного материала студентами и школьниками.. Решение уравнений онлайн. Уравнения онлайн. Различают алгебраические, параметрические, трансцендентные, функциональные, дифференциальные и другие виды уравнений.. Некоторые классы уравнений имеют аналитические решения, которые удобны тем, что не только дают точное значение корня, а позволяют записать решение в виде формулы, в которую могут входить параметры. Аналитические выражения позволяют не только вычислить корни, а провести анализ их существования и их количества в зависимости от значений параметров, что часто бывает даже важнее для практического применения, чем конкретные значения корней. Решение уравнений онлайн.. Уравнения онлайн. Решение уравнения — задача по нахождению таких значений аргументов, при которых это равенство достигается. На возможные значения аргументов могут быть наложены дополнительные условия (целочисленности, вещественности и т. д.). Решение уравнений онлайн.. Уравнения онлайн. Вы сможете решить уравнение онлайн моментально и с высокой точностью результата. Аргументы заданных функций (иногда называются «переменными») в случае уравнения называются «неизвестными». Значения неизвестных, при которых это равенство достигается, называются решениями или корнями данного уравнения. Про корни говорят, что они удовлетворяют данному уравнению. Решить уравнение онлайн означает найти множество всех его решений (корней) или доказать, что корней нет. Решение уравнений онлайн.. Уравнения онлайн. Равносильными или эквивалентными называются уравнения, множества корней которых совпадают. Равносильными также считаются уравнения, которые не имеют корней. Эквивалентность уравнений имеет свойство симметричности: если одно уравнение эквивалентно другому, то второе уравнение эквивалентно первому. Эквивалентность уравнений имеет свойство транзитивности: если одно уравнение эквивалентно другому, а второе эквивалентно третьему, то первое уравнение эквивалентно третьему. Свойство эквивалентности уравнений позволяет проводить с ними преобразования, на которых основываются методы их решения. Решение уравнений онлайн.. Уравнения онлайн. Сайт позволит решить уравнение онлайн. К уравнениям, для которых известны аналитические решения, относятся алгебраические уравнения, не выше четвёртой степени: линейное уравнение, квадратное уравнение, кубическое уравнение и уравнение четвёртой степени. Алгебраические уравнения высших степеней в общем случае аналитического решения не имеют, хотя некоторые из них можно свести к уравнениям низших степеней. Уравнения, в которые входят трансцендентные функции называются трансцендентными. Среди них аналитические решения известны для некоторых тригонометрических уравнений, поскольку нули тригонометрических функций хорошо известны. В общем случае, когда аналитического решения найти не удаётся, применяют численные методы. Численные методы не дают точного решения, а только позволяют сузить интервал, в котором лежит корень, до определённого заранее заданного значения. Решение уравнений онлайн.. Уравнения онлайн.. Вместо уравнения онлайн мы представим, как то же самое выражение образует линейную зависимость и не только по прямой касательной, но и в самой точке перегиба графика. Этот метод незаменим во все времена изучения предмета. Часто бывает, что решение уравнений приближается к итоговому значению посредством бесконечных чисел и записи векторов. Проверить начальные данные необходимо и в этом суть задания. Иначе локальное условие преобразуется в формулу. Инверсия по прямой от заданной функции, которую вычислит калькулятор уравнений без особой задержки в исполнении, взаимозачету послужит привилегия пространства. Речь пойдет о студентах успеваемости в научной среде. Впрочем, как и все вышесказанное, нам поможет в процессе нахождения и когда вы решите уравнение полностью, то полученный ответ сохраните на концах отрезка прямой. Линии в пространстве пересекаются в точке и эта точка называется пересекаемой линиями. Обозначен интервал на прямой как задано ранее. Высший пост на изучение математики будет опубликован. Назначить значению аргумента от параметрически заданной поверхности и решить уравнение онлайн сможет обозначить принципы продуктивного обращения к функции. Лента Мебиуса, или как её называет бесконечностью, выглядит в форме восьмерки. Это односторонняя поверхность, а не двухсторонняя. По принципу общеизвестному всем мы объективно примем линейные уравнения за базовое обозначение как есть и в области исследования. Лишь два значения последовательно заданных аргументов способны выявить направление вектора. Предположить, что иное решение уравнений онлайн гораздо более, чем просто его решение, обозначает получение на выходе полноценного варианта инварианта. Без комплексного подхода студентам сложно обучиться данному материалу. По-прежнему для каждого особого случая наш удобный и умный калькулятор уравнений онлайн поможет всем в непростую минуту, ведь достаточно лишь указать вводные параметры и система сама рассчитает ответ. Перед тем, как начать вводить данные, нам понадобится инструмент ввода, что можно сделать без особых затруднений. Номер каждой ответной оценки будет квадратное уравнение приводить к нашим выводам, но этого сделать не так просто, потому что легко доказать обратное. Теория, в силу своих особенностей, не подкреплена практическими знаниями. Увидеть калькулятор дробей на стадии опубликования ответа, задача в математике не из легких, поскольку альтернатива записи числа на множестве способствует увеличению роста функции. Впрочем, не сказать про обучение студентов было бы некорректным, поэтому выскажем каждый столько, сколько этого необходимо сделать. Раньше найденное кубическое уравнение по праву будет принадлежать области определения, и содержать в себе пространство числовых значений, а также символьных переменных. Выучив или зазубрив теорему, наши студенты проявят себя только с лучшей стороны, и мы за них будем рады. В отличие от множества пересечений полей, наши уравнения онлайн описываются плоскостью движения по перемножению двух и трех числовых объединенных линий. Множество в математике определяется не однозначно. Лучшее, по мнению студентов, решение — это доведенная до конца запись выражения. Как было сказано научным языком, не входит абстракция символьных выражений в положение вещей, но решение уравнений дает однозначный результат во всех известных случаях. Продолжительность занятия преподавателя складывается из потребностей в этом предложении. Анализ показал как необходимость всех вычислительных приемов во многих сферах, и абсолютно ясно, что калькулятор уравнений незаменимый инструментарий в одаренных руках студента. Лояльный подход к изучению математики обуславливает важность взглядов разных направленностей. Хотите обозначить одну из ключевых теорем и решите уравнение так, в зависимости от ответа которого будет стоять дальнейшая потребность в его применении. Аналитика в данной области набирает все мощный оборот. Начнем с начала и выведем формулу. Пробив уровень возрастания функции, линия по касательной в точке перегиба обязательно приведет к тому, что решить уравнение онлайн будет одним из главных аспектов в построении того самого графика от аргумента функции. Любительский подход имеет право быть применен, если данное условие не противоречит выводам студентов. На задний план выводится именно та подзадача, которая ставит анализ математических условий как линейные уравнения в существующей области определения объекта. Взаимозачет по направлению ортогональности взаимоуменьшает преимущество одинокого абсолютного значения. По модулю решение уравнений онлайн дает столько же решений, если раскрыть скобки сначала со знаком плюс, а затем со знаком минус. В таком случае решений найдется в два раза больше, и результат будет точнее. Стабильный и правильный калькулятор уравнений онлайн есть успех в достижении намеченной цели в поставленной преподавателем задаче. Нужный метод выбрать представляется возможным благодаря существенным отличиям взглядов великих ученых. Полученное квадратное уравнение описывает кривую линий так называемую параболу, а знак определит ее выпуклость в квадратной системе координат. Из уравнения получим и дискриминант, и сами корни по теореме Виета. Представить выражение в виде правильной или неправильной дроби и применить калькулятор дробей необходимо на первом этапе. В зависимости от этого будет складываться план дальнейших наших вычислений. Математика при теоретическом подходе пригодится на каждом этапе. Результат обязательно представим как кубическое уравнение, потому что его корни скроем именно в этом выражении, для того, чтобы упростить задачу учащемуся в ВУЗе. Любые методы хороши, если они пригодны к поверхностному анализу. Лишние арифметические действия не приведут к погрешности вычислений. С заданной точностью определит ответ. Используя решение уравнений, скажем прямо — найти независимую переменную от заданной функции не так-то просто, особенно в период изучения параллельных линий на бесконечности. В виду исключения необходимость очень очевидна. Разность полярностей однозначна. Из опыта преподавания в институтах наш преподаватель вынес главный урок, на котором были изучены уравнения онлайн в полном математическом смысле. Здесь речь шла о высших усилиях и особых навыках применения теории. В пользу наших выводов не стоит глядеть сквозь призму. До позднего времени считалось, что замкнутое множество стремительно возрастает по области как есть и решение уравнений просто необходимо исследовать. На первом этапе мы не рассмотрели все возможные варианты, но такой подход обоснован как никогда. Лишние действия со скобками оправдывают некоторые продвижения по осям ординат и абсцисс, чего нельзя не заметить невооруженным глазом. В смысле обширного пропорционального возрастания функции есть точка перегиба. В лишний раз докажем как необходимое условие будет применяться на всем промежутке убывания той или иной нисходящей позиции вектора. В условиях замкнутого пространства мы выберем переменную из начального блока нашего скрипта. За отсутствие главного момента силы отвечает система, построенная как базис по трем векторам. Однако калькулятор уравнений вывел, и помогло в нахождении всех членов построенного уравнения, как над поверхностью, так и вдоль параллельных линий. Вокруг начальной точки опишем некую окружность. Таким образом, мы начнем продвигаться вверх по линиям сечений, и касательная опишет окружность по всей ее длине, в результате получим кривую, которая называется эвольвентой. Кстати расскажем об этой кривой немного истории. Дело в том, что исторически в математике не было понятия самой математики в чистом понимании как сегодня. Раньше все ученые занимались одним общим делом, то есть наукой. Позже через несколько столетий, когда научный мир наполнился колоссальным объемом информации, человечество все-таки выделило множество дисциплин. Они до сих пор остались неизменными. И все же каждый год ученые всего мира пытаются доказать, что наука безгранична, и вы не решите уравнение, если не будете обладать знаниями в области естественных наук. Окончательно поставить точку не может быть возможным. Об этом размышлять также бессмысленно, как согревать воздух на улице. Найдем интервал, на котором аргумент при положительном своем значении определит модуль значения в резко возрастающем направлении. Реакция поможет отыскать как минимум три решения, но необходимо будет проверить их. Начнем с того, что нам понадобиться решить уравнение онлайн с помощью уникального сервиса нашего сайта. Введем обе части заданного уравнения, нажмем на кнопу «РЕШИТЬ» и получим в течение всего нескольких секунд точный ответ. В особых случаях возьмем книгу по математике и перепроверим наш ответ, а именно посмотрим только ответ и станет все ясно. Вылетит одинаковый проект по искусственному избыточному параллелепипеду. Есть параллелограмм со своими параллельными сторонами, и он объясняет множество принципов и подходов к изучению пространственного отношения восходящего процесса накопления полого пространства в формулах натурального вида. Неоднозначные линейные уравнения показывают зависимость искомой переменной с нашим общим на данный момент времени решением и надо как-то вывести и привести неправильную дробь к нетривиальному случаю. На прямой отметим десять точек и проведем через каждую точку кривую в заданном направлении, и выпуклостью вверх. Без особых трудностей наш калькулятор уравнений представит в таком виде выражение, что его проверка на валидность правил будет очевидна даже в начале записи. Система особых представлений устойчивости для математиков на первом месте, если иного не предусмотрено формулой. На это мы ответим подробным представление доклада на тему изоморфного состояния пластичной системы тел и решение уравнений онлайн опишет движение каждой материальной точки в этой системе. На уровне углубленного исследования понадобится подробно выяснить вопрос об инверсиях как минимум нижнего слоя пространства. По возрастанию на участке разрыва функции мы применим общий метод великолепного исследователя, кстати, нашего земляка, и расскажем ниже о поведении плоскости. В силу сильных характеристик аналитически заданной функции, мы используем только калькулятор уравнений онлайн по назначению в выведенных пределах полномочий. Рассуждая далее, остановим свой обзор на однородности самого уравнения, то есть правая его часть приравнена к нулю. Лишний раз удостоверимся в правильности принятого нами решения по математике. Во избежание получения тривиального решения, внесем некоторые корректировки в начальные условия по задаче на условную устойчивость системы. Составим квадратное уравнение, для которого выпишем по известной всем формуле две записи и найдем отрицательные корни. Если один корень на пять единиц превосходит второй и третий корни, то внесением правок в главный аргумент мы тем самым искажаем начальные условия подзадачи. По своей сути нечто необычное в математике можно всегда описать с точностью до сотых значений положительного числа. В несколько раз калькулятор дробей превосходит свои аналоги на подобных ресурсах в самый лучший момент нагрузки сервера. По поверхности растущего по оси ординат вектора скорости начертим семь линий, изогнутых в противоположные друг другу направления. Соизмеримость назначенного аргумента функции опережает показания счетчика восстановительного баланса. В математике этот феномен представим через кубическое уравнение с мнимыми коэффициентами, а также в биполярном прогрессе убывания линий. Критические точки перепада температуры во много своем значении и продвижении описывают процесс разложения сложной дробной функции на множители. Если вам скажут решите уравнение, не спешите это делать сию минуту, однозначно сначала оцените весь план действий, а уже потом принимайте правильный подход. Польза будет непременно. Легкость в работе очевидна, и в математике то же самое. Решить уравнение онлайн. Все уравнения онлайн представляют собой определенного вида запись из чисел или параметров и переменной, которую нужно определить. Вычислить эту самую переменную, то есть найти конкретные значения или интервалы множества значений, при которых будет выполняться тождество. Напрямую зависят условия начальные и конечные. В общее решение уравнений как правило входят некоторые переменные и константы, задавая которые, мы получим целые семейства решений для данной постановки задачи. В целом это оправдывает вкладываемые усилия по направлению возрастания функциональности пространственного куба со стороной равной 100 сантиметрам. Применить теорему или лемму можно на любом этапе построения ответа. Сайт постепенно выдает калькулятор уравнений при необходимости на любом интервале суммирования произведений показать наименьшее значение. В половине случаев такой шар как полый, не в большей степени отвечает требованиям постановки промежуточного ответа. По крайней мере на оси ординат в направлении убывания векторного представления эта пропорция несомненно будет являться оптимальнее предыдущего выражения. В час, когда по линейным функциям будет проведен полный точечный анализ, мы, по сути, соберем воедино все наши комплексные числа и биполярные пространства плоскостной. Подставив в полученное выражение переменную, вы решите уравнение поэтапно и с высокой точностью дадите максимально развернутый ответ. Лишний раз проверить свои действия в математике будет хорошим тоном со стороны учащегося студента. Пропорция в соотношении дробей зафиксировала целостность результата по всем важным направлениям деятельности нулевого вектора. Тривиальность подтверждается в конце выполненных действий. С простой поставленной задачей у студентов не может возникнуть сложностей, если решить уравнение онлайн в самые кратчайшие периоды времени, но не забываем о всевозможных правилах. Множество подмножеств пересекается в области сходящихся обозначений. В разных случаях произведение не ошибочно распадается на множители. Решить уравнение онлайн вам помогут в нашем первом разделе, посвященном основам математических приемов для значимых разделов для учащихся в ВУЗах и техникумах студентов. Ответные примеры нас не заставят ожидать несколько дней, так как процесс наилучшего взаимодействия векторного анализа с последовательным нахождением решений был запатентован в начале прошлого века. Выходит так, что усилия по взаимосвязям с окружающим коллективом были не напрасными, другое очевидно назрело в первую очередь. Спустя несколько поколений, ученые всего мира заставили поверить в то, что математика это царица наук. Будь-то левый ответ или правый, все равно исчерпывающие слагаемые необходимо записать в три ряда, поскольку в нашем случае речь пойдет однозначно только про векторный анализ свойств матрицы. Нелинейные и линейные уравнения, наряду с биквадратными уравнениями, заняли особый пост в нашей книге про наилучшие методы расчета траектории движения в пространстве всех материальных точек замкнутой системы. Воплотить идею в жизнь нам поможет линейный анализ скалярного произведения трех последовательных векторов. В конце каждой постановки, задача облегчается благодаря внедрениям оптимизированных числовых исключений в разрез выполняемых наложений числовых пространств. Иное суждение не противопоставит найденный ответ в произвольной форме треугольника в окружности. Угол между двумя векторами заключает в себе необходимый процент запаса и решение уравнений онлайн зачастую выявляет некий общий корень уравнения в противовес начальным условиям. Исключение выполняет роль катализатора во всем неизбежном процессе нахождения положительного решения в области определения функции. Если не сказано, что нельзя пользоваться компьютером, то калькулятор уравнений онлайн в самый раз подойдет для ваших трудных задач. Достаточно лишь вписать в правильном формате свои условные данные и наш сервер выдаст в самые кратчайшие сроки полноценный результирующий ответ. Показательная функция возрастает гораздо быстрее, чем линейная. Об этом свидетельствую талмуды умной библиотечной литературы. Произведет вычисление в общем смысле как это бы сделало данное квадратное уравнение с тремя комплексными коэффициентами. Парабола в верхней части полуплоскости характеризует прямолинейное параллельное движение вдоль осей точки. Здесь стоит упомянуть о разности потенциалов в рабочем пространстве тела. Взамен неоптимальному результату, наш калькулятор дробей по праву занимает первую позицию в математическом рейтинге обзора функциональных программ на серверной части. Легкость использования данного сервиса оценят миллионы пользователей сети интернет. Если не знаете, как им воспользоваться, то мы с радостью вам поможем. Еще хотим особо отметить и выделить кубическое уравнение из целого ряда первостепенных школьнических задач, когда необходимо быстро найти его корни и построить график функции на плоскости. Высшие степени воспроизведения — это одна из сложных математических задач в институте и на ее изучение выделяется достаточное количество часов. Как и все линейные уравнения, наши не исключение по многих объективным правилам, взгляните под разными точками зрений, и окажется просто и достаточно выставить начальные условия. Промежуток возрастания совпадает с интервалом выпуклости функции. Решение уравнений онлайн. В основе изучения теории состоят уравнения онлайн из многочисленных разделов по изучению основной дисциплины. По случаю такого подхода в неопределенных задачах, очень просто представить решение уравнений в заданном заранее виде и не только сделать выводы, но и предсказать исход такого положительного решения. Выучить предметную область поможет нам сервис в самых лучших традициях математики, именно так как это принято на Востоке. В лучшие моменты временного интервала похожие задачи множились на общий множитель в десять раз. Изобилием умножений кратных переменных в калькулятор уравнений завелось приумножать качеством, а не количественными переменными таких значений как масса или вес тела. Во избежание случаев дисбаланса материальной системы, нам вполне очевиден вывод трехмерного преобразователя на тривиальном схождении невырожденных математических матриц. Выполните задание и решите уравнение в заданных координатах, поскольку вывод заранее неизвестен, как и неизвестны все переменные, входящие в пост пространственное время. На короткий срок выдвинете общий множитель за рамки круглых скобок и поделите на наибольший общий делитель обе части заранее. Из-под получившегося накрытого подмножества чисел извлечь подробным способом подряд тридцать три точки за короткий период. Постольку поскольку в наилучшем виде решить уравнение онлайн возможно каждому студенту, забегая вперед, скажем одну важную, но ключевую вещь, без которой в дальнейшем будем непросто жить. В прошлом веке великий ученый подметил ряд закономерностей в теории математики. На практике получилось не совсем ожидаемое впечатление от событий. Однако в принципе дел это самое решение уравнений онлайн способствует улучшению понимания и восприятия целостного подхода к изучению и практическому закреплению пройдённого теоретического материала у студентов. На много проще это сделать в свое учебное время.

=

Что такое уравнение и в чем его смысл?

Вообще любое уравнение — это математическая модель чашечных весов (рычажных, равноплечих, коромысловых — названий много), изобретенных в древнем Вавилоне 7000 лет назад или еще раньше. Более того, я даже думаю, что именно чашечные весы, использовавшиеся на древнейших базарах, и стали прообразом уравнений. И если смотреть на любое уравнение не как на непонятный набор цифр и букв, связанный двумя параллельными палочками, а как на чаши весов, то и со всем остальным проблем не будет:

Любое уравнение подобно уравновешенным чашам весов

Так уж получилось, что уравнений в нашей жизни с каждым днем все больше, а понимания, что такое уравнение и в чем его смысл — все меньше. Во всяком случае у меня сложилось такое впечатление при попытке объяснить старшей дочери смысл простейшего математического уравнения типа:

х + 2 = 8 (500.1)

Т.е. в школе конечно же объясняют, что в таких случаях чтобы найти х, нужно из правой части вычесть 2:

х = 8 — 2 (500.3)

Это, конечно же, абсолютно правильное действие, но почему нужно именно вычесть, а не, например, прибавить или разделить, в школьных учебниках объяснения нет. Просто есть правило, которое нужно тупо выучить:

При переносе члена уравнения из одной части в другую его знак меняется на противоположный.

А как сие правило понимать школьнику 10 лет от роду и в чем его смысл, это вы уж сами думайте-решайте. Более того, выяснилось, что и мои близкие родственники тоже никогда не понимали смысла уравнений, а просто заучивали на память то, что требовалось (и вышеуказанное правило в частности), а уж потом применяли это, как бог на душу положит. Мне такое положение дел не понравилось, поэтому я и решил написать данную статью (растет младший, ему через несколько лет опять придется это объяснять, да и немногочисленным читателям моего сайта это тоже может пригодиться).

Сразу хочу сказать, что хоть я 10 лет учился в школе, но при этом никаких правил и определений, относящихся к техническим дисциплинам, никогда не учил. Т.е. если что-то понятно, то оно и так запомнится, а если что-то не понятно, то какой смысл его зубрить, не понимая смысла, если оно все равно забудется? А кроме того, если мне что-то не понятно, значит, оно мне и не надо (это я только недавно осознал, что если я чего-то не понимал в школе, то это была не моя вина, а вина преподавателей, учебников и вообще системы образования).

Такой подход обеспечивал мне массу свободного времени, которого в детстве так не хватает на всякие игры и развлечения. При этом я участвовал в различных олимпиадах по физике, химии, а одну районную по математике даже выиграл. Но время шло, количество дисциплин, оперирующих абстрактными понятиями, только увеличивалось и соответственно мои оценки снижались. На первом курсе института количество дисциплин, оперирующих абстрактными понятиями, составляло абсолютное большинство и я конечно же был полным троечником. Но потом, когда мне по ряду причин пришлось самому без помощи лекций и конспектов разбираться с сопроматом и я его как бы понял, дело пошло на лад и закончилось красным дипломом. Впрочем сейчас не об этом, а о том, что в связи с указанной спецификой мои понятия и определения могут значительно отличаться от преподаваемых в школе.

А теперь продолжим

Простейшие уравнения, аналогия с весами

Вообще-то детей приучают сравнивать различные предметы еще в дошкольном возрасте, когда они еще и говорить-то толком не умеют. Начинают как правило с геометрических сравнений. Например, показывают ребенку два кубика и ребенок должен определить, какой кубик больше, а какой меньше. А если они одинаковые, то это и есть равенство по размеру. Затем задача усложняется, ребенку показывают предметы различных форм, различных цветов и выбрать одинаковые предметы ребенку становится все сложнее. Однако мы не будем так сильно усложнять задачу, а остановимся лишь на одном виде равенства — денежно-весовом.

Когда чаши весов находятся на одном горизонтальном уровне (стрелки чашечных весов, показанные на рисунке 500.1 оранжевым и голубым цветом, совпадают, горизонтальный уровень показан черной жирной чертой), то это значит, что на правой чаше весов находится столько же груза, сколько на левой чаше. В простейшем случае это могут быть гири весом в 1 кг:

Рисунок 500.1.

И тогда мы получаем простейшее уравнение 1 = 1. Впрочем уравнение это только для меня, в математике подобные выражения называют равенством, но суть от этого не меняется. Если мы с левой чаши весов уберем гирю и положим на нее что угодно, хоть яблоки, хоть гвозди, хоть красную икру и при этом чаши весов будут на одном горизонтальном уровне, то это будет по-прежнему означать, что 1 кг любого из указанных продуктов равен 1 кг гирьки, оставшейся на правой части весов. Остается лишь заплатить за этот килограмм согласно установленной продавцом цене. Другое дело, что вам может не нравиться цена, или возникли сомнения в точности весов — но это уже вопросы экономико-правовых отношений, к математике прямого отношения не имеющие.

Конечно же, в те далекие времена, когда появились чашечные весы, все было значительно проще. Во-первых, не было такой меры веса, как килограмм, а были денежные единицы, соответствующие мерам весов, например, таланты, шекели, фунты, гривны и пр. (кстати, меня давно удивляло, что есть фунт — денежная единица и фунт — мера веса, есть гривна — денежная единица, а когда-то гривна была мерой веса, и только недавно, когда я узнал, что талант — это не только денежная единица древних иудеев, упоминаемая в Ветхом завете, но и мера веса, принятая в древнем Вавилоне, все встало на свои места).

Точнее сначала были меры весов, как правило зерна злаковых культур, а уже потом появились деньги, этим мерам весов соответствующие. Например 60 зерен соответствовали одному шекелю (сиклю), 60 шекелей — одной мине, а 60 мин — одному таланту. Поэтому изначально весы использовались для того, чтобы проверить, не являются ли предлагаемые деньги фальшивыми, а уже потом появились гирьки, как эквивалент денег, обвесы и обсчеты, электронные весы и пластиковые карты, но сути дела это никак не меняет.

В те далекие времена продавцу не нужно было долго и подробно объяснять, сколько будет стоить тот или иной товар. Достаточно было положить на одну чашу весов продаваемый товар, а на вторую покупатель клал деньги — очень просто и наглядно и даже знание местного наречия не требуется, можно торговать в любой точке мира. Но вернемся к уравнениям.

Если рассматривать уравнение (500.1) с позиции весов, то оно означает, что на левой чаше весов находится неизвестное количество килограммов и еще 2 килограмма, а на правой чаше — 8 килограммов:

х + 2кг, = 8кг, (500.1.2)

Примечание: В данном случае нижнее подчеркивание символизирует дно чаш весов, при расчетах на бумаге эта линия может больше напоминать дно чаши весов. Более того, математики уже давно придумали специальные символы — скобки, так вот любые скобки можно рассматривать как борта чаш весов, во всяком случае на первом этапе постижения смысла уравнений. Тем не менее нижнее подчеркивание я для большей наглядности оставлю.

Итак, что нам нужно сделать, что узнать неизвестное количество килограммов? Правильно! Снять с левой и с правой части весов по 2 килограмма, тогда чаши весов останутся на одном горизонтальном уровне, т.е.у нас будет по прежнему равенство:

х + 2кг, — 2кг = 8кг, — 2кг (500.2.2)

Соответственно

х, = 8кг — 2кг, (500.3.2)

х, = 6 кг, (500.4.2)

Рисунок 500.2.

Часто математика оперирует не килограммами, а некими абстрактными безразмерными единицами и тогда запись решения уравнения (500.1) например в черновике будет выглядеть так:

х + 2, = 8, (500.1)

х + 2, — 2 = 8, — 2 (500.2)

х, = 8 — 2, (500.3)

х = 6 (500.4)

Что и отражено на рисунке 500.2.

Примечание: Формально для еще более лучшего понимания  после уравнения (500.2) должно следовать еще одно уравнение вида: х + 2 — 2, = 8 — 2, означающее, что действие завершилось и мы опять имеем дело с равновесными чашами весом. Однако на мой взгляд в такой совсем уж полной записи решения необходимости нет.

В чистовиках обычно используется сокращенная запись решения уравнения, причем сокращаются не только столь необходимые на мой взгляд на начальном этапе изучения уравнений символы чаш весов, но даже и целые уравнения. Так сокращенная запись решения уравнения (500.1) в чистовике согласно приводимым в учебниках примерам будет выглядеть так:

х + 2 = 8 (500.1.1)

х = 8 — 2 (500.3.1)

х = 6 (500.4)

В итоге, при использовании аналогии с весами мы составили дополнительное уравнение (500.2) по сравнению с предлагаемым учебниками то ли методом решения, то ли формой записи этого решения. На мой взгляд это уравнение, к тому же записанное приблизительно в такой форме, т.е. с символичным обозначением чаш весов — это и есть то недостающее звено, важное для понимания смысла уравнений.

Т.е. при решении уравнений мы ничего и никуда с обратным знаком не переносим, а выполняем одинаковые математические действия с левой и с правой частью уравнения.

Просто сейчас принято записывать решение уравнений в сокращенной форме, приведенной выше. За уравнением (500.1.1) сразу следует уравнение (500.3.1), отсюда и следует правило обратных знаков, которое впрочем многим проще запомнить, чем вникать в смысл уравнений.

Примечание: Против сокращенной формы записи я ничего не имею, более того. продвинутые пользователи могут эту форму еще более сокращать, однако делать это следует лишь после того, когда общий смысл уравнений уже четко усвоен.

А еще расширенная запись позволяет понять главные правила решения уравнений:

1. Если мы производим одинаковые математические действия с левой и правой частью уравнений, то равенство сохраняется.

2. Не важно, какая часть в рассматриваемом уравнении левая, а какая правая, мы можем свободно менять их местами.

Эти математические действия могут быть любыми. Мы можем вычитать одно и то же число из левой и из правой части, как показано выше. Мы можем прибавлять одно и то же число к левой и правой части уравнения, например:

х — 2, = 8, (500.5.1)

х — 2, + 2 = 8, + 2 (500.5.2)

х, = 8 + 2, (500.5.3)

х  = 10 (500.5.4)

Мы можем делить или умножать обе части на одно и то же число, например:

3х, = 12, (500.6.1)

3х, : 3 = 12, : 3 (500.6.2)

х, = 12 : 3, (500.6.3)

х  = 4 (500.6.4)

или

3х — 6, = 12, (500.7.1)

3х — 6, + 6 = 12, + 6 (500.7.2)

3х, = 18, (500.7.3)

3х, : 3 = 18, : 3 (500.7.4)

х = 6 (500.7.5)

Мы можем интегрировать или дифференцировать обе части. Мы можем делать все, что угодно с левой и правой частью, но если эти действия будут одинаковыми для левой и правой части, то равенство сохранится (чаши весов останутся на одном горизонтальном уровне).

Конечно же действия нужно выбирать такие, которые позволят максимально быстро и просто определить неизвестную величину.

С этой точки зрения классический метод обратного действия как бы более прост, но как быть, если ребенок еще не изучал отрицательные числа? А между тем составленное уравнение имеет следующий вид:

5 — х = 3 (500.8)

Т.е. при решении этого уравнения классическим методом один из возможных вариантов решения, дающий самую короткую запись, следующий:

— х = 3 — 5 (500.8.2)

— х = — 2 (500.8.3)

х = 2 (500.8.4)

И самое главное — как тут объяснить ребенку почему уравнение (500.8.3) тождественно уравнению (500.8.4)?

Это значит, что в данном случае даже при использовании классического метода экономить на записи нет никакого смысла и сначала нужно избавиться от неизвестной величины в левой части, имеющей отрицательный знак.

5 — х = 3 (500.8)

5 = 3 + х (500.8.5)

3 + х = 5 (500.8.6)

х = 5 — 3 (500.8.7)

х = 2 (500.8.4)

При этом полная запись будет выглядеть так:

5 — х, = 3, (500.8)

5 — х, + х = 3, + х (500.9.2)

5, = 3 + х, (500.9.3)

3 + х, = 5, (500.8.6)

3 + х, — 3 = 5, — 3 (500.9.3)

х, = 5 — 3, (500.8.7)

х = 2 (500.8.4)

Добавлю еще раз. Полная запись решения нужна не для учителей, а для лучшего понимания метода решения уравнений. А когда мы меняем местами левую и правую части уравнения, то это все равно что мы меняем взгляд на весы с точки зрения покупателя на точку зрения продавца, тем не менее равенство при этом сохраняется.

К сожалению, я так и не смог добиться от своей дочери полной записи решения даже в черновиках. У нее железный довод: «нас так не учили». А между тем сложность составляемых уравнений увеличивается, процент угадываний, какое действие нужно выполнить для определения неизвестной величины, уменьшается, оценки падают. Что с этим делать, не знаю…

Примечание: в современной математике принято различать равенства и уравнения, т.е. 1 = 1 — это просто численное равенство, а если в одной из частей равенства есть неизвестная, которую необходимо найти, то это уже уравнение. Как по мне, то такое дифференцирование значений не имеет большого смысла, а лишь усложняет восприятие материала. Я считаю, что любое равенство можно называть уравнением, а любое уравнение основано на равенстве. А кроме того, возникает вопрос х = 6, это уже равенство или это еще уравнение?

Простейшие уравнения, аналогия со временем

Конечно же, аналогия с весами при решении уравнений является далеко не единственной. Например, решение уравнений можно рассматривать и во временном аспекте. Тогда условие, описываемое уравнением (500.1), будет звучать так:

После того, как мы добавили к неизвестному количеству х еще 2 единицы, у нас стало 8 единиц (настоящее время). Однако нас по тем или иным причинам не интересует, сколько их стало, а интересует сколько их было в прошедшем времени. Соответственно, чтобы узнать, сколько у нас было этих самых единиц, нам нужно произвести обратное действие, т.е. от 8 отнять 2 (уравнение 500.3). Такой подход точно соответствует излагаемому в учебниках, но на мой взгляд, является не таким наглядным, как аналогия с весами. Впрочем мнения по этому поводу могут быть разные.

Пример решения уравнения со скобками

Эту статью я написал летом, когда дочь окончила 4 класс, но не прошло и полгода, как им в школе начали задавать решение уравнений следующего вида:

(97 + 75 : (50 — 5х)) · 3 = 300 (500.10)

Никто в классе решить это уравнение не смог, а между тем в его решении при применении предложенного мной способа нет ничего сложного, вот только полная форма записи будет занимать слишком много места:

(97 + 75 : (50 — 5х)) · 3, : 3 = 300, : 3 (500.10.2)

97 + 75 : (50 — 5х), = 300 : 3, (500.10.3)

97 + 75 : (50 — 5х), = 100, (500.10.4)

97 + 75 : (50 — 5х), — 97 =  100, — 97 (500.10.5)

75 : (50 — 5х), = 100 — 97, (500.10.6)

75 : (50 — 5х), = 3, (500.10.7)

75 : (50 — 5х), · (50 — 5х) = 3, · (50 — 5х) (500.10.8)

75, = 3 · (50 — 5х), (500.10.9)

75, : 3 = 3 · (50 — 5х), : 3 (500.10.10)

75 : 3, = 50 — 5х, (500.10.11)

25, = 50 — 5х, (500.10.12)

25, + 5х = 50 — 5х, + 5х (500.10.13)

25 + 5х, = 50, (500.10.14)

25 + 5х, — 25 = 50, — 25 (500.10.15)

5х, = 50 — 25, (500.10.16)

5х, = 25, (500.10.17)

5х, : 5 = 25, : 5 (500.10.18)

х, = 25 : 5, (500.10.19)

х = 5 (500.10.20)

Однако на данном этапе в такой полной форме записи нет никакой необходимости. Раз уж мы добрались до двойных скобок, то не обязательно для математических операций в левой и правой части составлять отдельное уравнение, поэтому запись решения в черновике вполне может выглядеть так:

97 + 75 : (50 — 5х), : 3 = 300, : 3, (500.10.2)

97 + 75 : (50 — 5х), = 100, (500.10.4)

97 + 75 : (50 — 5х), — 97 =  100 — 97, (500.10.5)

75 : (50 — 5х), = 3, (500.10.7)

75 : (50 — 5х), · (50 — 5х) = 3, · (50 — 5х) (500.10.8)

75, = 3 · (50 — 5х), (500.10.9)

75, : 3 = 3 · (50 — 5х), : 3 (500.10.10)

25, = 50 — 5х, (500.10.12)

25, + 5х = 50 — 5х, + 5х (500.10.13)

25 + 5х, = 50, (500.10.14)

25 + 5х, — 25 = 50, — 25 (500.10.15)

5х, = 25, (500.10.17)

5х, : 5 = 25, : 5 (500.10.18)

х = 5 (500.10.20)

Итого на данном этапе потребовалось записать 14 уравнений для решения исходного.

При этом запись решения уравнения в чистовике может выглядеть так:

97 + 75 : (50 — 5х) = 300 : 3 (500.10.3)

97 + 75 : (50 — 5х) = 100 (500.10.4)

75 : (50 — 5х) = 100 — 97 (500.10.6)

75 : (50 — 5х) = 3 (500.10.7)

75 = 3 · (50 — 5х) (500.10.9)

75 : 3 = 50 — 5х (500.10.11)

25 = 50 — 5х (500.10.12)

25 + 5х = 50 (500.10.14)

5х = 50 — 25 (500.10.16)

5х = 25 500.10.17)

х = 25 : 5 (500.10.19)

х = 5 (500.10.20)

Т.е. при сокращенной форме записи нам все равно придется составить 12 уравнений. Экономия в записи при этом минимальная, а вот с пониманием требуемых действий у пятиклассника действительно могут возникнуть проблемы.

P.S. Только когда дело дошло до двойных скобок, дочь заинтересовалась предложенным мной методом решения уравнений, но при этом в ее форме записи даже в черновике все равно уравнений в 2 раза меньше, потому что она пропускает итоговые уравнения типа (500.10.4), (500.10.7) и им подобные, а при записи сразу оставляет место для следующего математического действия. В итоге запись в ее черновике выглядела примерно так:

(97 + 75 : (50 — 5х)) · 3, : 3 = 300, : 3 (500.10.2)

97 + 75 : (50 — 5х), — 97 =  100, — 97 (500.10.5)

75 : (50 — 5х), · (50 — 5х) = 3, · (50 — 5х) (500.10.8)

75, : 3 = 3 · (50 — 5х), : 3 (500.10.10)

25, + 5х = 50 — 5х, + 5х (500.10.13)

25 + 5х, — 25 = 50, — 25 (500.10.15)

5х, : 5 = 25, : 5 (500.10.18)

х = 5 (500.10.20)

В итоге получилось всего 8 уравнений, что даже меньше, чем требуется при сокращенной записи решения. В принципе я не возражаю, вот только была бы от этого польза.

Вот собственно и все, что мне хотелось сказать по поводу решения простейших уравнений, содержащих одну неизвестную величину. Для решения уравнений, содержащих две неизвестных величины, потребуется больше знаний.

«Решение уравнений» (урок рефлексии по учебнику Л.Г.Петерсон «Математика. 1-й класс»)

Учебник: Л.Г. Петерсон «Математика. 1 класс».

Урок рефлексии.

Цели:

  1. Закрепить умение решать уравнения на вычитание, основанные на взаимодействии части и целого.
  2. Развивать математическую речь, логическое мышление, умение работать по алгоритму.
  3.  Воспитывать коммуникативные качества, через работу в парах.

Оборудование:

  1.  Карточки для закрепления быстрого правильного счета в пределах 9.
  2.  Карточки для самостоятельной работы №1, №2 с образцом для самопроверки.
  3.  Карточки с алгоритмом комментирования решения уравнений.
  4.  Схемы для составления уравнений.

Ход урока

I. Орг. момент.

  • Упражнение для активизации мыслительной деятельности «Ленивые восьмёрки».

Чтобы наша работа на уроке шла успешно, проведём упражнение.

Нарисуйте в воздухе «лежачую восьмёрку» по три раза каждой рукой, а затем обеими руками.

  • Приветствие гостей.

Мы рады приветствовать вас в классе нашем.
Возможно, есть классы и лучше и краше.
Но пусть в нашем классе вам будет светло.
Пусть будет уютно и очень легко.
Поручено нам вас сегодня встречать.
Ну что ж, начнём урок, не будем зря время терять

II. Самоопределение к деятельности.

  1. Чему вы учитесь на уроках математики?
  2. Вы сказали правильно писать цифры.
  3. Зачем? Где вам может пригодиться?
  4. Сейчас мы поупражняемся в правильном написании чисел.
  5. Зарядка для пальчиков «Котятки».

У котяток лапки царапки, (сжимание и разжимание кулачков)
Любят котятки лапками царапать (сжали кулачки).
Выпустят один коготок, выпустят второй коготок,
Третий, четвёртый, пятый (поочерёдное разжимание пальчиков, начиная с большёго).
И ну давай царапать (разжимание и сжимание кулачков).

– На доске записан ряд чисел.

– Что вы можете о них сказать? (однозначные, чётные)

– Установите закономерность (увеличиваются на 2).

– Продолжите ряд на 2 числа (8, 10) — проверка . Какое число лишнее? Почему? Самостоятельно повторите ряд 1 раз.

  • Чему вы сейчас учились? Где это вам может пригодиться?
  • Вы сказали, что на уроках математики учитесь быстро и правильно читать.
  • Что для этого необходимо знать? Уметь?

Проверим, как вы быстро и правильно умеете считать в пределах 9.

9-8 = У

6-2 = В

5-3 = Р

8-5=А

5+4=Е

6+2=И

3 = 3= Е

7-2 = Н

4= 3 = Н

  • Что у вас получилось? (выражения)
  •  Что записано вместо ответов? (буквы)

Перед вами на партах лежат листочки с табличками (покажите их).

1

2

3

4

5

6

7

8

9

 

 

 

 

 

 

 

 

 

Вы должны найти значение выражений и под ответами записать буквы в клеточки, соответственно ответам.

Если, вы, правильно найдёте значение выражений,у вас получится слово (2 мин).

(Проверка).

– Какое слово у вас получилось? (уравнение)

Значит, состав чисел в пределах 9 вы знаете хорошо, и не будете допускать ошибки в вычислениях.

  • У кого не получилось слово?
  • Почему?
  • Над чем нужно поработать?

III. Актуализация знаний.

  • Объясните, что такое уравнение?
  • Что в уравнении может быть неизвестным?
  • Давайте вспомним, как найти целое? Часть?

Какую цель поставите перед собой?

Чему будете учиться на уроке? (решать уравнения на вычитание).

Попробуйте назвать тему {Решение уравнений на вычитание)

Физминутка

IV. Фиксирование затруднен.

  1. Работа в парах (Запись уравнений на файлах) 1,3 ряд составляют уравнения на вычитание по схеме

  1. 2 ряд составляет уравнение на вычитание по схеме:

Выделите части и целое.

  • Как работаем в паре?
  • Как показываем готовность?

Ученики крепят файлы под схемами, объясняют.

  • Кто не согласен? Почему?

Решение уравнений.

Кто помнит, какую цель поставили? Чему будем учиться?

  • Чтобы решать уравнения, что необходимо знать? (правила, алгоритм)

Алгоритм решения уравнений:

  1. Найти в уравнении части и целое.
  2. Определить, чем является х, частью или целым.
  3. Применить соответствующее правило.
  4. Назвать и записать ответ.
  5.  Сделать проверку.

> Правила мы повторили, давайте повторим алгоритм (алгоритм вывешен на доске).

А сейчас, пользуясь алгоритмом, попробуйте решить уравнения самостоятельно по вариантам (ученики на обратной стороне доски)

Проверка.

Объяснение по алгоритму?

Алгоритм комментирования решения уравнений:

  1. Читаю уравнение:
  2. В этом уравнении части… и целое — …
  3. Применяю правило: …
  4. х равен сумме (разности) … и …
  5. Ответ: х равен …
  6. Проверяю: …

Кто не согласен, стукните карандашом.

V. Самостоятельная работа с проверкой по образцу.

Сейчас вы самостоятельно попробуете решить уравнение, будете сами себя проверять и оценивать.

У каждого из вас лежат на партах карточки с самостоятельными работами № 1 и № 2.

  • Вы выполняете сначала самостоятельную работу № 1 первое уравнение.
  • Затем проверяете себя по образцу, отгибая правый край карточки, если допустили ошибки, выполняете второе уравнение.
  • Если нет, переходите к самостоятельной работе № 2, проверяете по образцу.
  • Если выполнили, устно составляете задачу по схеме, с помощью которых вы составляли уравнения, эти схемы находятся на доске.
  • Проверку выполняете зеленой ручкой, у кого нет, простым карандашом.

– Повторите, как поняли задание.

– Когда можно открывать образец? (когда выполнили задание)

– Почему? (Потому, что каждый должен научиться решать такие уравнения сам).

VI. Рефлексия

  • Какую цель ставили перед собой?
  • А теперь себя оцените.
  • Как оцениваем? (Eсли всё получилось, рисуем себе зелёный кружок; если, я, старался, но у меня не всё получилось, жёлтый; мне нужна помощь – красный.)
  • Поставьте себе нужный кружок.

VII. Итог.

  • Поднимите руку те, у кого зелёный кружок? Молодцы! Значит уравнения такого вида, вы, научились решать.
  • Жёлтый? Как, вы, думаете, почему? Над чем нужно поработать?
  • Красный? Как, вы, думаете, почему? Над чем нужно поработать?

Домашнее задание: составить и решить два уравнения на вычитание.

Приложение. Карточки для самостоятельной работы с самопроверкой по образцу.

Решайте уравнения и упрощайте выражения (Алгебра 2, Уравнения и неравенства) — Mathplanet

В алгебре 1 нас учат, что два правила решения уравнений — это правило сложения и правило умножения / деления.
Правило сложения для уравнений говорит нам, что одна и та же величина может быть добавлена ​​к обеим сторонам уравнения без изменения набора решений уравнения.


Пример

$$ \ begin {array} {lcl} 4x-12 & = & 0 \\ 4x-12 + 12 & = & 0 + 12 \\ 4x & = & 12 \\ \ end {array} $$

Добавление 12 к каждой стороне уравнения в первой строке примера — это первый шаг в решении уравнения.Мы не меняли решение, добавляя по 12 с каждой стороны, поскольку и второе, и третье уравнения имеют одно и то же решение. Уравнения, которые имеют одинаковые наборы решений, называются эквивалентными уравнениями.

Правило умножения / деления для уравнений говорит нам, что каждый член в обеих частях уравнения может быть умножен или разделен на один и тот же член (кроме нуля) без изменения набора решений уравнения.


Пример

$$ \ begin {array} {lcl} 4x-12 & = & 0 \\ 4x-12 + 12 & = & 0 + 12 \\ 4x & = & 12 \\ \ frac {4x} {4} & = & \ frac {12} {4} \\ x & = & 3 \\ \ end {array} $$

Когда мы упрощаем выражение, мы действуем в следующем порядке:

  1. Упростите выражения внутри круглых скобок, скобок, фигурных скобок и дробей.{2} -2)} {\ sqrt {2}} $$

    Сначала мы упрощаем выражение в круглых скобках, вычисляя степени, а затем выполняем вычитание внутри него.

    $$ \ frac {(4-2)} {\ sqrt {2}} $$

    $$ \ frac {(2)} {\ sqrt {2}} $$

    Затем мы убираем скобки и умножаем знаменатель и числитель на √2.

    $$ \ frac {2 \ cdot \ sqrt {2}} {\ sqrt {2} \ cdot \ sqrt {2}} $$

    В качестве последнего шага мы делаем все умножения и деления слева направо.

    $$ \ frac {2 \ cdot \ sqrt {2}} {2} $$

    $$ \ sqrt {2} $$


    Видеоурок

    Решите данное уравнение

    $$ 12 (\ frac {3b-b} {4a}) = 36 $$

    Решение линейных уравнений — Полный курс алгебры

    9

    Закон обратного

    Четыре формы уравнений

    Транспонирование

    Логическая последовательность операторов

    Транспонирование и обмен сторон

    Форма ax = 0

    Раздел 2 :

    Отмена

    Неизвестное с обеих сторон

    Простые дробные уравнения

    УРАВНЕНИЕ — это алгебраическое утверждение, в котором глагол «равно» =.Уравнение включает неизвестное число, обычно называемое x . Вот простой пример:

    x + 4 = 10.

    «Некоторое число плюс 4 равно 10.»

    Мы говорим, что уравнение имеет две стороны: левую, x + 4, и правую, 10.

    Поскольку x появляется в первой степени, мы называем это линейным уравнением. Линейное уравнение еще называют уравнением первой степени.

    Степень любого уравнения — это наивысший показатель степени неизвестного числа. Уравнение первой степени называется линейным , потому что, как мы увидим много позже, его график представляет собой прямую линию .

    Уравнение — это утверждение — станет истинным только тогда, когда неизвестное имеет определенное значение, которое мы называем решением уравнения.

    Решение этого уравнения, очевидно, 6:

    6 + 4 = 10.

    6 — единственное значение x , для которого верно утверждение « x + 4 = 10». Мы говорим, что x = 6 удовлетворяет уравнению.

    Итак, алгебра зависит от того, как все выглядит. Что касается того, как все выглядит, то мы узнаем, что решили уравнение, когда выделим x слева.

    Почему налево? Потому что мы читаем слева направо. « x равно.. . «

    В стандартной форме линейного уравнения — ax + b = 0 — x появляется слева.

    Фактически, мы видели, что для любого уравнения, которое выглядит так:

    x + a = б ,
    решение всегда будет выглядеть так:
    x = b a .
    Если
    x + 4 = 10,
    , затем
    x = 10 — 4
    = 6.

    Закон обратного

    Есть две пары обратных операций. Сложение и вычитание, умножение и деление.

    Формально, чтобы решить уравнение, мы должны изолировать неизвестное на одной стороне уравнения.

    ax b + c = d .

    Мы должны переместить a, b , c на другую сторону, так что x будет один.

    Вопрос:

    Как перенести число из одной части уравнения
    в другую?

    Ответ:

    Путем записи на другой стороне с помощью обратной операции.

    Это закон обратного. Это следует из двух Правил 5 урока.

    Пример 1. Решите это уравнение:

    a x — b + c = д .
    Решение. Поскольку b — это , вычтенное из слева, мы будем прибавлять к справа:
    a x + c = д + б .
    Так как c — это , добавленное слева, мы вычтем справа:
    ось = d + b c .
    И, наконец, поскольку a умножает слева, мы будем разделить справа:
    x = d + b c
    a

    Мы решили уравнение.

    Четыре формы уравнений

    Таким образом, решение любого линейного уравнения распадается на четыре формы, соответствующие четырем операциям арифметики.Ниже приведены основные правила решения любого линейного уравнения. В каждом случае мы будем перемещать на на другую сторону.

    1. Если x + a = b , то x = b — a .

    «Если число , добавленное с одной стороны уравнения,
    мы можем вычесть с другой стороны».

    2. Если x a = b , то x = b + a .

    «Если число вычтено из на одной стороне уравнения,
    мы можем прибавить на другой стороне».

    3. Если ax = b , то x = b
    a
    .

    «Если число умножает на одну сторону уравнения,
    мы можем разделить его на на другую сторону.«

    4. Если x
    a
    = b , тогда x = ab .

    «Если число делит на одну сторону уравнения,
    мы можем умножить на на другую сторону».

    В каждом случае a были сдвинуты на другую сторону посредством обратной операции.Любое линейное уравнение можно будет решить, применив одно или несколько из этих правил.

    Транспонирование

    Когда используются операции сложения или вычитания (формы 1 и 2), мы называем это транспонированием.

    Мы можем переместить член на другую сторону уравнения
    , изменив его знак на .

    + a переходит на другую сторону как — a .

    a переходит на другую сторону как + a .

    Транспонирование — одна из наиболее характерных операций алгебры, и считается, что это значение слова алгебра , имеющего арабское происхождение. (Арабские математики изучали алгебру в Индии, откуда они представили ее в Европе.) Транспонирование — это техника тех, кто действительно использует алгебру в науке и математике — потому что это искусно. И, как мы скоро увидим, в нем сохраняется четкая логическая последовательность утверждений. Более того, это подчеркивает, что вы занимаетесь алгеброй глазами.Когда вы видите

    x + a = б ,
    тогда вы сразу видите , что + a переходит на другую сторону как — a :
    x = b a .

    Однако часто учат писать — a с обеих сторон, начертить линию и сложить.

    Во-первых, вы никогда не увидите этого ни в одном математическом тексте. Вы увидите логическую последовательность утверждений, к которой мы скоро подойдем.

    Более того, мы доказали, что можем просто транспонировать. Нет необходимости доказывать это снова каждый раз, когда вы решаете уравнение.

    (Вам нужно доказывать теорему Пифагора каждый раз, когда вы ее применяете? Нет, вы этого не сделаете.)

    Если вы хотите представить, что вы вычли из с обеих сторон, хорошо. Но писать приходиться не умело.

    Вот что вы увидите в своем тексте расчетов.

    Логическая последовательность операторов

    Рассмотрим снова уравнение из Примера 1.

    ax b + c = d .

    Это алгебраическое предложение — это утверждение — логически подразумевает другие утверждения.Теперь мы увидим логическую последовательность, которая приводит к окончательному утверждению, которое и является решением.

    (1) топор b + c = д
    подразумевает (2) топор = d + b c
    подразумевает (3) x = d + b c .
    а

    Исходное уравнение (1) «преобразуется» путем перестановки членов. Из утверждения (1) следует утверждение (2).

    Затем этот оператор преобразуется делением на и . Из утверждения (2) следует утверждение (3), которое является решением.

    Таким образом, мы решаем уравнение, преобразуя его — изменяя его внешний вид — оператор за оператором, строка за строкой в ​​соответствии с правилами алгебры, пока наконец не будет выделено x слева.Так пишут книги по математике (но, к сожалению, не книги по алгебре!). Каждая строка представляет собой собственное читаемое утверждение, которое следует из строки выше — без зачеркивания.

    Другими словами, что такое расчет? Это дискретное преобразование символов. В арифметике преобразовываем «19 + 5» в «24». В алгебре мы преобразуем « x + a = b » в « x = b a ».

    Проблема 1.Напишите логическую последовательность операторов, которая решит это уравнение для x :

    .

    abcx d + e f = 0

    Чтобы увидеть ответ, наведите указатель мыши на
    слева направо над цветной областью.
    Чтобы закрыть ответ еще раз, нажмите «Обновить» («Reload»).
    Сначала решите проблему сами!

    (1) abcx d + e f = 0
    подразумевает (2) abcx = d e + f
    подразумевает (3) x = d e + f .
    abc

    Сначала транспонируйте термины . Строка (2).

    Нет необходимости писать справа термин 0.

    Затем разделите на коэффициент x .

    Задача 2. Напишите логическую последовательность операторов, которая решит это уравнение для x :

    (1) 2 x + 5 = 27
    подразумевает (2) 2 x = 27-5 = 22
    подразумевает (3) x = 22
    2
    подразумевает (4) x = 11.

    В задачах 3, 4 и 5 дано только решение. Студент должен написать логическую последовательность утверждений, которая к нему приводит.

    Задача 3. Решите для x : ( p q ) x + r = s

    Задача 4. Решите для x :

    ab ( c + d ) x e + f = 0

    x = e f
    ab ( c + d )

    Проблема 5.Решите относительно x : 2 x + 1 = 0

    х = −½

    Каждое из приведенных выше уравнений имеет стандартную форму, а именно:

    ax + b = 0.

    a не означает a . Значит коэффициент х . И b не означает b . Это означает любые термины.

    Вот почему он называется формой.Как бы то ни было, выглядит так .

    Проблема 6. Решить: топор + b = 0.
    x = b
    a

    Это простое уравнение иллюстрирует выполнение алгебры глазами. учащийся должен немедленно увидеть решение. Вы должны увидеть , что b перейдет на другую сторону как — b , и что a разделит.

    Это навык в алгебре.

    Задача 7. Решите для x : ax = 0 ( a 0).

    Теперь, когда произведение двух чисел равно 0, то хотя бы одно из них должно быть 0. (Урок 6.) Следовательно, любое уравнение с такой формой имеет решение:

    х = 0.

    Мы могли бы решить это формально, конечно, разделив на и .

    Задача 8. Решите для x :

    4 x -2 = −2
    4 x = −2 + 2 = 0
    x = 0.

    Задача 9. Напишите последовательность операторов, которая решит это уравнение:

    (1) 6 — x = 9
    (2) х = 9–6
    (3) х = 3
    (4) x = −3.

    Когда мы переходим от строки (1) к строке (2), — x остается слева. Для, члены в строке (1) равны 6 и — x .

    Мы «решили» уравнение, когда изолировали слева x , а не x . Поэтому мы переходим от строки (3) к строке (4), меняя знаки с обеих сторон. (Урок 5.)

    В качестве альтернативы мы могли исключить — x слева, сразу поменяв все знаки:

    (1) 6 — x = 9
    (2) −6 + х = −9
    (3) x = −9 + 6 = −3.
    Задача 10. Решите для x : 3 — x = −5
    x = 8.

    Проблема 11.Решить относительно x :

    4 — (2 x — 1) = −11.
    4-2 x + 1 = −11.
    5-2 x = −11
    −2 x = −11-5
    2 x = 16
    x = 8.

    Задача 12. Решите для x :

    3 x -15
    2x + 1
    = 0.

    ( Подсказка : Сравните Урок 6, Задачу 18.)

    x = 5.

    Транспонирование и обмен сторон

    Пример 2. a + b = c x

    Мы можем легко решить эту проблему — в одной строке — просто переместив x влево, а то, что слева, вправо:

    x = c a b .

    Пример 3. a + b = c + x

    В этом примере + x справа.Так как нам нужно + x слева, мы можем добиться этого, поменяв стороны:

    c + x = a + b

    Примечание: При обмене сторонами знаки не меняются.

    После транспонирования c легко следует решение:

    c + x = a + b c .

    Таким образом, когда — x справа, его можно просто транспонировать.Но когда + x справа, мы можем поменять стороны.

    Задача 13. Решите для x :

    p + q = r x s
    Транспонировать:
    x = r s p q

    Проблема 14.Решить относительно x :

    p q + r = с + x
    Сменные стороны:
    с + x = p q + r
    x = p q + r s

    Проблема 15.Решить относительно x :

    0 = пикселей + q
    пикселей + q = 0
    пикселей = q
    x = q
    p

    Проблема 16.Решить относительно x :

    -2 = −5 x + 1
    5 x = 1 + 2 = 3
    x = 3
    5

    Проблема 17.Решить относительно x :

    п. = q ax .
    ось = q p
    x = q p
    a

    Проблема 18.Решите относительно cos θ («косинус против -ta»).

    Стоит увидеть, что это уравнение имеет точно такую ​​же форму и , что и задача 17. cos θ — это неизвестное. Вы решите ее точно так же, как задачу 17.

    2 cos θ = 8 — А
    cos θ = 8 — А
    2

    Алгебра состоит в распознавании формы.И их только конечное число.

    Раздел 2 :

    Отмена

    Неизвестное с обеих сторон

    Простые дробные уравнения

    Содержание | Дом


    Авторские права © 2021 Лоуренс Спектор

    Вопросы или комментарии?

    Эл. Почта: [email protected]


    Использование правила Крамера для решения системы двух уравнений с двумя переменными

    Вычисление определителя матрицы 2 × 2

    Определитель — это действительное число, которое может быть очень полезно в математике, потому что у него есть несколько приложений, таких как вычисление площади, объема и других величин.Здесь мы будем использовать определители, чтобы определить, является ли матрица обратимой, используя элементы квадратной матрицы , чтобы определить, существует ли решение системы уравнений. Однако, возможно, одним из наиболее интересных приложений является их использование в криптографии. Защищенные сигналы или сообщения иногда отправляются в виде матрицы. Расшифровать данные можно только с помощью обратимой матрицы и определителя. В наших целях мы ориентируемся на определитель как на показатель обратимости матрицы.Вычисление определителя матрицы требует следования определенным шаблонам, описанным в этом разделе.

    Общее примечание: Найдите определитель матрицы 2 × 2

    Определитель матрицы [latex] 2 \ text {} \ times \ text {} 2 [/ latex], учитывая

    [латекс] A = \ left [\ begin {array} {cc} a & b \\ c & d \ end {array} \ right] [/ latex]

    определяется как

    Рисунок 1

    Обратите внимание на изменение обозначений. Есть несколько способов указать определитель, включая [latex] \ mathrm {det} \ left (A \ right) [/ latex] и замену скобок в матрице прямыми линиями, [latex] | A | [/ latex] .

    Пример 1: Нахождение определителя матрицы 2 × 2

    Найдите определитель заданной матрицы.

    [латекс] A = \ left [\ begin {array} {cc} 5 & 2 \\ -6 & 3 \ end {array} \ right] [/ latex]

    Решение

    [латекс] \ begin {array} {l} \ mathrm {det} \ left (A \ right) = | \ begin {array} {cc} 5 & 2 \\ -6 & 3 \ end {array} | \ hfill \ \ = 5 \ left (3 \ right) — \ left (-6 \ right) \ left (2 \ right) \ hfill \\ = 27 \ hfill \ end {array} [/ latex]

    Использование правила Крамера для решения системы двух уравнений с двумя переменными

    Теперь мы представим последний метод решения систем уравнений, использующий определители.Этот метод, известный как правило Крамера , восходит к середине 18 века и назван в честь своего новатора, швейцарского математика Габриэля Крамера (1704–1752), который представил его в 1750 году во Введении к анализу линий Курб. algébriques. Правило Крамера — это жизнеспособный и эффективный метод поиска решений систем с произвольным числом неизвестных, при условии, что у нас есть такое же количество уравнений, что и неизвестных.

    Правило Крамера даст нам единственное решение системы уравнений, если оно существует.Однако, если система не имеет решения или бесконечное количество решений, это будет обозначено нулевым определителем. Чтобы выяснить, является ли система непоследовательной или зависимой, необходимо использовать другой метод, например исключение.

    Чтобы понять правило Крамера, давайте внимательно рассмотрим, как мы решаем системы линейных уравнений с использованием основных операций со строками. Рассмотрим систему двух уравнений с двумя переменными.

    [латекс] \ begin {array} {c} {a} _ {1} x + {b} _ {1} y = {c} _ {1} \ left (1 \ right) \\ {a} _ { 2} x + {b} _ {2} y = {c} _ {2} \ left (2 \ right) \ end {array} [/ latex]

    Мы исключаем одну переменную, используя операции со строками, и решаем для другой.Скажите, что мы хотим решить для [latex] x [/ latex]. Если уравнение (2) умножается на коэффициент, противоположный коэффициенту [латекс] y [/ латекс] в уравнении (1), уравнение (1) умножается на коэффициент [латекс] y [/ латекс] в уравнении (2) ), и мы добавляем два уравнения, переменная [latex] y [/ latex] будет удалена.

    [латекс] \ begin {array} \ text {} b_ {2} a_ {1} x + b_ {2} b_ {1} y = b_ {2} c_ {1} \ hfill & \ text {Multiply} R_ { 1} \ text {by} b_ {2} \\ — b_ {1} a_ {2} x − b_ {1} b_ {2} y = −b_ {1} c_ {2} \ hfill & \ text {Умножить} R_ {2} \ text {by} −b_ {2} \\ \ text {______________________} \\ b_ {2} a_ {1} x − b_ {1} a_ {2} x = −b_ {2} c_ { 1} −b_ {1} c_ {2} \ end {array} [/ latex]

    Теперь решите для [латекс] x [/ латекс].

    [латекс] \ begin {array} {l} {b} _ {2} {a} _ {1} x- {b} _ {1} {a} _ {2} x = {b} _ {2 } {c} _ {1} — {b} _ {1} {c} _ {2} \ hfill \\ x \ left ({b} _ {2} {a} _ {1} — {b} _ {1} {a} _ {2} \ right) = {b} _ {2} {c} _ {1} — {b} _ {1} {c} _ {2} \ hfill \\ \ text { } x = \ frac {{b} _ {2} {c} _ {1} — {b} _ {1} {c} _ {2}} {{b} _ {2} {a} _ {1 } — {b} _ {1} {a} _ {2}} = \ frac {\ left [\ begin {array} {cc} {c} _ {1} & {b} _ {1} \\ { c} _ {2} & {b} _ {2} \ end {array} \ right]} {\ left [\ begin {array} {cc} {a} _ {1} & {b} _ {1} \\ {a} _ {2} & {b} _ {2} \ end {array} \ right]} \ hfill \ end {array} [/ latex]

    Аналогичным образом, чтобы найти [latex] y [/ latex], мы исключим [latex] x [/ latex].

    [латекс] \ begin {array} \ text {} a_ {2} a_ {1} x + a_ {2} b_ {1} y = a_ {2} c_ {1} \ hfill & \ text {Multiply} R_ { 1} \ text {by} a_ {2} \\ — a_ {1} a_ {2} x − a_ {1} b_ {2} y = −a_ {1} c_ {2} \ hfill & \ text {Умножить} R_ {2} \ text {by} −a_ {1} \\ \ text {______________________} \\ a_ {2} b_ {1} y − a_ {1} b_ {2} y = a_ {2} c_ {1 } −a_ {1} c_ {2} \ end {array} [/ latex]

    Решение для [latex] y [/ latex] дает

    [латекс] \ begin {array} {l} {a} _ {2} {b} _ {1} y- {a} _ {1} {b} _ {2} y = {a} _ {2 } {c} _ {1} — {a} _ {1} {c} _ {2} \ hfill \\ y \ left ({a} _ {2} {b} _ {1} — {a} _ {1} {b} _ {2} \ right) = {a} _ {2} {c} _ {1} — {a} _ {1} {c} _ {2} \ hfill \\ \ text { } y = \ frac {{a} _ {2} {c} _ {1} — {a} _ {1} {c} _ {2}} {{a} _ {2} {b} _ {1 } — {a} _ {1} {b} _ {2}} = \ frac {{a} _ {1} {c} _ {2} — {a} _ {2} {c} _ {1} } {{a} _ {1} {b} _ {2} — {a} _ {2} {b} _ {1}} = \ frac {| \ begin {array} {cc} {a} _ { 1} & {c} _ {1} \\ {a} _ {2} & {c} _ {2} \ end {array} |} {| \ begin {array} {cc} {a} _ {1 } & {b} _ {1} \\ {a} _ {2} & {b} _ {2} \ end {array} |} \ hfill \ end {array} [/ latex]

    Обратите внимание, что знаменатель для [latex] x [/ latex] и [latex] y [/ latex] является определителем матрицы коэффициентов.

    Мы можем использовать эти формулы для решения для [latex] x [/ latex] и [latex] y [/ latex], но правило Крамера также вводит новые обозначения:

    Ключом к правилу Крамера является замена интересующего столбца переменных столбцом констант и вычисление детерминантов. Затем мы можем выразить [латекс] x [/ latex] и [latex] y [/ latex] как частное двух определителей.

    Общее примечание: правило Крамера для систем 2 × 2

    Правило Крамера — это метод, который использует детерминанты для решения систем уравнений, которые имеют то же количество уравнений, что и переменные.

    Рассмотрим систему двух линейных уравнений с двумя переменными.

    [латекс] \ begin {array} {c} {a} _ {1} x + {b} _ {1} y = {c} _ {1} \\ {a} _ {2} x + {b} _ {2} y = {c} _ {2} \ end {array} [/ latex]

    Решение, использующее правило Крамера, дается как

    [латекс] x = \ frac {{D} _ {x}} {D} = \ frac {| \ begin {array} {cc} {c} _ {1} & {b} _ {1} \\ {c} _ {2} & {b} _ {2} \ end {array} |} {| \ begin {array} {cc} {a} _ {1} & {b} _ {1} \\ { a} _ {2} & {b} _ {2} \ end {array} |}, D \ ne 0; \ text {} \ text {} y = \ frac {{D} _ {y}} {D } = \ frac {| \ begin {array} {cc} {a} _ {1} & {c} _ {1} \\ {a} _ {2} & {c} _ {2} \ end {array } |} {| \ begin {array} {cc} {a} _ {1} & {b} _ {1} \\ {a} _ {2} & {b} _ {2} \ end {array} |}, D \ ne 0 [/ латекс].

    Если мы решаем для [latex] x [/ latex], столбец [latex] x [/ latex] заменяется столбцом констант. Если мы решаем для [latex] y [/ latex], столбец [latex] y [/ latex] заменяется постоянным столбцом.

    Пример 2: Использование правила Крамера для решения системы 2 × 2

    Решите следующую систему [latex] 2 \ text {} \ times \ text {} 2 [/ latex], используя правило Крамера.

    [латекс] \ begin {array} {c} 12x + 3y = 15 \\ \ text {} 2x — 3y = 13 \ end {array} [/ latex]

    Решение

    Решите для [латекс] x [/ латекс].

    [латекс] x = \ frac {{D} _ {x}} {D} = \ frac {| \ begin {array} {rr} \ hfill 15 & \ hfill 3 \\ \ hfill 13 & \ hfill -3 \ end {array} |} {| \ begin {array} {rr} \ hfill 12 & \ hfill 3 \\ \ hfill 2 & \ hfill -3 \ end {array} |} = \ frac {-45 — 39} {- 36 — 6} = \ frac {-84} {- 42} = 2 [/ latex]

    Найдите [латекс] и [/ латекс].

    [латекс] y = \ frac {{D} _ {y}} {D} = \ frac {| \ begin {array} {rr} \ hfill 12 & \ hfill 15 \\ \ hfill 2 & \ hfill 13 \ end { array} |} {| \ begin {array} {rr} \ hfill 12 & \ hfill 3 \\ \ hfill 2 & \ hfill -3 \ end {array} |} = \ frac {156 — 30} {- 36 — 6} = — \ frac {126} {42} = — 3 [/ латекс]

    Решение [латекс] \ left (2, -3 \ right) [/ latex].

    Попробуйте 1

    Используйте правило Крамера для решения системы уравнений 2 × 2.

    [латекс] \ begin {массив} {l} \ text {} x + 2y = -11 \ hfill \\ -2x + y = -13 \ hfill \ end {array} [/ latex]

    9.8: Решение систем с помощью правила Крамера

    Мы узнали, как решать системы уравнений с двумя переменными и тремя переменными и с помощью нескольких методов: подстановки, сложения, исключения Гаусса, использования обратной матрицы и построения графиков. Некоторые из этих методов применять проще, чем другие, и они более подходят в определенных ситуациях.В этом разделе мы изучим еще две стратегии решения систем уравнений.

    Вычисление определителя матрицы 2 × 2

    Определитель — это действительное число, которое может быть очень полезно в математике, потому что у него есть несколько приложений, таких как вычисление площади, объема и других величин. Здесь мы будем использовать определители, чтобы определить, является ли матрица обратимой, используя элементы квадратной матрицы, чтобы определить, существует ли решение системы уравнений.Однако, возможно, одним из наиболее интересных приложений является их использование в криптографии. Защищенные сигналы или сообщения иногда отправляются в виде матрицы. Расшифровать данные можно только с помощью обратимой матрицы и определителя. В наших целях мы ориентируемся на определитель как на показатель обратимости матрицы. Вычисление определителя матрицы требует следования определенным шаблонам, описанным в этом разделе.

    НАЙТИ ОПРЕДЕЛЕНИЕ МАТРИЦЫ 2 × 2

    Определитель матрицы 2 × 2, учитывая

    \ (A = \ begin {bmatrix} a & b \\ c & d \ end {bmatrix} \)

    определяется как

    Обратите внимание на изменение обозначений.Есть несколько способов указать определитель, включая \ (\ det (A) \) и замену скобок в матрице прямыми линиями, \ (| A | \).

    Пример \ (\ PageIndex {1} \): поиск определителя матрицы \ (2 × 2 \)

    Найдите определитель заданной матрицы.

    \ (A = \ begin {bmatrix} 5 & 2 \\ — 6 & 3 \ end {bmatrix} \)

    Решение

    \ [\ begin {align *} \ det (A) & = \ begin {vmatrix} 5 & 2 \\ — 6 & 3 \ end {vmatrix} \\ & = 5 (3) — (- 6) (2) \\ & = 27 \ end {align *} \]

    Использование правила Крамера для решения системы двух уравнений с двумя переменными

    Теперь мы представим последний метод решения систем уравнений, использующий определители.Этот метод, известный как правило Крамера , восходит к середине 18 века и назван в честь своего новатора, швейцарского математика Габриэля Крамера (1704-1752), который представил его в 1750 году в году. Courbes algébriques . Правило Крамера — это жизнеспособный и эффективный метод поиска решений систем с произвольным числом неизвестных, при условии, что у нас есть такое же количество уравнений, что и неизвестных.

    Правило Крамера даст нам единственное решение системы уравнений, если оно существует.Однако, если система не имеет решения или бесконечное количество решений, это будет обозначено нулевым определителем. Чтобы выяснить, является ли система непоследовательной или зависимой, необходимо использовать другой метод, например исключение.

    Чтобы понять правило Крамера, давайте внимательно рассмотрим, как мы решаем системы линейных уравнений с использованием основных операций со строками. Рассмотрим систему двух уравнений с двумя переменными.

    \ [\ begin {align} a_1x + b_1y & = c_1 (1) \ label {eq1} \\ a_2x + b_2y & = c_2 (2) \ label {eq2} \\ \ end {align} \]

    Мы исключаем одну переменную, используя операции со строками, и решаем для другой.Скажите, что мы хотим найти \ (x \). Если уравнение \ ref {eq2} умножается на коэффициент, противоположный коэффициенту \ (y \) в уравнении \ ref {eq1}, уравнение \ ref {eq1} умножается на коэффициент при \ (y \) в уравнении \ ref {eq2}, и мы добавляем два уравнения, переменная \ (y \) будет удалена.

    \ [\ begin {align *} & b_2a_1x + b_2b_1y = b_2c_1 & \ text {Multiply} R_1 \ text {by} b_2 \\ — & \ underline {b_1a_2x − b_1b_2y = −b_1c_2} & \ text {Multiply} R_2 \ text {by} −b_1 \\ & b_2a_1x − b_1a_2x = b_2c_1 − b_1c_2 \ end {align *} \]

    Теперь решите относительно \ (x \).

    \ [\ begin {align *} b_2a_1x − b_1a_2x & = b_2c_1 − b_1c_2 \\ x (b_2a_1 − b_1a_2) & = b_2c_1 − b_1c_2 \\ x & = \ dfrac {b_2c_1 − b_1c_2} {b_2a_1 − b_1a_2} = \ dfrac {\ begin {bmatrix} c_1 & b_1 \\ c_2 & b_2 \ end {bmatrix}} {\ begin {bmatrix} a_1 & b_1 \\ a_2 & b_2 \ end {bmatrix}} \ end {align *} \]

    Аналогично, чтобы найти \ (y \), мы исключим \ (x \).

    \ [\ begin {align *} & a_2a_1x + a_2b_1y = a_2c_1 & \ text {Multiply} R_1 \ text {by} a_2 \\ — & \ underline {a_1a_2x − a_1b_2y = −a_1c_2} & \ text {Multiply} R_2 \ текст {by} −a_1 \\ & a_2b_1y − a_1b_2y = a_2c_1 − a_1c_2 \ end {align *} \]

    Решение относительно \ (y \) дает

    \ [\ begin {align *} a_2b_1y − a_1b_2y & = a_2c_1 − a_1c_2 \\ y (a_2b_1 − a_1b_2) & = a_2c_1 − a_1c_2 \\ y & = \ dfrac {a_2c_1 − a_1c_2} {a_2b_1 − a_1b_2} = \ dfrac {a_1c_2 − a_2c_1} {a_1b_2 − a_2b_1} = \ dfrac {\ begin {bmatrix} a_1 & c_1 \\ a_2 & c_2 \ end {bmatrix}} {\ begin {bmatrix} a_1 & b_1 \\ a_2 & b_2 \ end {bmatrix}} \ end {align * } \]

    Обратите внимание, что знаменатель для \ (x \) и \ (y \) является определителем матрицы коэффициентов.

    Мы можем использовать эти формулы для решения относительно \ (x \) и \ (y \), но правило Крамера также вводит новые обозначения:

    • \ (D \): определитель матрицы коэффициентов
    • \ (D_x \): определитель числителя в решении \ (x \)

      \ [x = \ dfrac {D_x} {D} \]

    • \ (D_y \): определитель числителя в решении \ (y \)

      \ [y = \ dfrac {D_y} {D} \]

    Ключ к правилу Крамера — заменить интересующий столбец переменных столбцом констант и вычислить детерминанты.Тогда мы можем выразить \ (x \) и \ (y \) как частное двух определителей.

    ПРАВИЛО КРЕМЕРА ДЛЯ СИСТЕМ \ (2 × 2 \)

    Правило Крамера — это метод, использующий детерминанты для решения систем уравнений, которые имеют то же количество уравнений, что и переменные.

    Рассмотрим систему двух линейных уравнений с двумя переменными.

    \ [\ begin {align *} a_1x + b_1y & = c_1 \\ a_2x + b_2y & = c_2 \ end {align *} \]

    Решение, использующее правило Крамера, дается как

    \ [\ begin {align} x & = \ dfrac {D_x} {D} = \ dfrac {\ begin {bmatrix} c_1 & b_1 \\ c_2 & b_2 \ end {bmatrix}} {\ begin {bmatrix} a_1 & b_1 \\ a_2 & b_2 \ end { bmatrix}} \; , D \ neq 0 \\ y & = \ dfrac {D_y} {D} = \ dfrac {\ begin {bmatrix} a_1 & c_1 \\ a_2 & c_2 \ end {bmatrix}} {\ begin {bmatrix} a_1 & b_1 \\ a_2 & b_2 \ end {bmatrix }} \; , D \ neq 0 \ end {align} \]

    Если мы решаем для \ (x \), столбец \ (x \) заменяется постоянным столбцом.Если мы решаем для \ (y \), столбец \ (y \) заменяется постоянным столбцом.

    Пример \ (\ PageIndex {2} \): использование правила Крамера для решения системы \ (2 × 2 \)

    Решите следующую систему \ (2 × 2 \), используя правило Крамера.

    \ [\ begin {align *} 12x + 3y & = 15 \\ 2x-3y & = 13 \ end {align *} \]

    Решение

    Решите относительно \ (x \).

    \ [\ begin {align *} x & = \ dfrac {D_x} {D} \\ & = \ dfrac {\ begin {bmatrix} 15 & 3 \\ 13 & -3 \ end {bmatrix}} {\ begin {bmatrix} 12 & 3 \\ 2 & -3 \ end {bmatrix}} \\ & = \ dfrac {-45-39} {- 36-6} \\ & = \ dfrac {-84} {- 42} \\ & = 2 \ end {align *} \]

    Решите относительно \ (y \).

    \ [\ begin {align *} y & = \ dfrac {D_y} {D} \\ & = \ dfrac {\ begin {bmatrix} 12 & 15 \\ 2 & 13 \ end {bmatrix}} {\ begin {bmatrix} 12 & 3 \\ 2 & -3 \ end {bmatrix}} \\ & = \ dfrac {156-30} {- 36-6} \\ & = — \ dfrac {126} {42} \\ & = -3 \ end {align * } \]

    Решение: \ ((2, −3) \).

    Упражнение \ (\ PageIndex {1} \)

    Используйте правило Крамера для решения системы уравнений \ (2 × 2 \).

    \ [\ begin {align *} x + 2y & = -11 \\ -2x + y & = -13 \ end {align *} \]

    Ответ

    \ ((3, −7) \)

    Вычисление определителя матрицы 3 × 3

    Найти определитель матрицы 2 × 2 несложно, но найти определитель матрицы 3 × 3 сложнее.Один из способов — увеличить матрицу 3 × 3 повторением первых двух столбцов, получив матрицу 3 × 5. Затем мы вычисляем сумму произведений записей на по каждой из трех диагоналей (от верхнего левого угла к нижнему правому) и вычитаем произведение записей на по каждой из трех диагоналей (нижний левый верхний правый). Это легче понять с помощью наглядного пособия и примера.

    Найдите определитель матрицы 3 × 3.

    \ (A = \ begin {bmatrix} a_1 & b_1 & c_1 \\ a_2 & b_2 & c_2 \\ a_3 & b_3 & c_3 \ end {bmatrix} \)

    1. Дополнение \ (A \) с первыми двумя столбцами.

      \ (\ det (A) = \ left | \ begin {array} {ccc | cc} a_1 & b_1 & c_1 & a_1 & b_1 \\ a_2 & b_2 & c_2 & a_2 & b_2 \\ a_3 & b_3 & c_3 & a_3 & b_3 \ end {array} \ right | \)

    2. С верхнего левого угла в нижний правый: умножение значений по первой диагонали. Добавьте результат к произведению записей по второй диагонали. Добавьте этот результат к произведению записей по третьей диагонали.
    3. От левого нижнего угла до правого верхнего: вычтите произведение значений первой диагонали.Из этого результата вычтите произведение входов вверх по второй диагонали. Из этого результата вычтите произведение входов до третьей диагонали.

    Алгебра выглядит следующим образом:

    \ (| A | = a_1b_2c_3 + b_1c_2a_3 + c_1a_2b_3 − a_3b_2c_1 − b_3c_2a_1 − c_3a_2b_1 \)

    Пример \ (\ PageIndex {3} \): поиск определителя матрицы 3 × 3

    Найдите определитель матрицы \ (3 × 3 \) при

    \ (A = \ begin {bmatrix} 0 & 2 & 1 \\ 3 & −1 & 1 \\ 4 & 0 & 1 \ end {bmatrix} \)

    Решение

    Дополните матрицу первыми двумя столбцами, а затем следуйте формуле.Таким образом,

    \ [\ begin {align *} | А | & = \ left | \ begin {array} {ccc | cc} 0 & 2 & 1 & 0 & 2 \\ 3 & -1 & 1 & 3 & -1 \\ 4 & 0 & 1 & 4 & 0 \ end {array} \ right | \\ & = 0 (−1) (1) +2 (1) (4) +1 (3) (0) −4 (−1) (1) −0 (1) (0) −1 (3) (2) \\ & = 0 + 8 + 0 + 4−0−6 \\ & = 6 \ end {align *} \]

    Упражнение \ (\ PageIndex {2} \)

    Найдите определитель матрицы 3 × 3.

    \ (\ det (A) = \ begin {vmatrix} 1 & −3 & 7 \\ 1 & 1 & 1 \\ 1 & −2 & 3 \ end {vmatrix} \)

    Ответ

    \ (- 10 \)

    Q&A: Можем ли мы использовать тот же метод, чтобы найти определитель большей матрицы?

    Нет, этот метод работает только для матриц 2 × 2 и 3 × 3.Для больших матриц лучше всего использовать графическую утилиту или компьютерное программное обеспечение.

    Использование правила Крамера для решения системы трех уравнений с тремя переменными

    Теперь, когда мы можем найти определитель матрицы \ (3 × 3 \), мы можем применить правило Крамера для решения системы трех уравнений с тремя переменными. Правило Крамера простое и соответствует шаблону, соответствующему правилу Крамера для матриц \ (2 × 2 \). Однако по мере увеличения порядка матрицы до \ (3 × 3 \) требуется гораздо больше вычислений.

    Когда мы вычисляем, что определитель равен нулю, правило Крамера не дает никаких указаний на то, что у системы нет решения или есть бесконечное количество решений. Чтобы выяснить это, мы должны выполнить устранение в системе.

    Рассмотрим систему уравнений \ (3 × 3 \).

    \ [\ begin {align} a_1x + b_1y + c_1z & = \ color {blue} d_1 \\ a_2x + b_2y + c_2z & = \ color {blue} d_2 \\ a_3x + b_3y + c_3z & = \ color {blue} d_3 \\ \ end {align} \]

    \ (x = \ dfrac {D_x} {D} \), \ (y = \ dfrac {D_y} {D} \), \ (z = \ dfrac {D_z} {D} \), \ (D ≠ 0 \)

    где

    \ [D = \ begin {vmatrix} a_1 & b_1 & c_1 \\ a_2 & b_2 & c_2 \\ a_3 & b_3 & c_3 \ end {vmatrix} \; , \; D_x = \ begin {vmatrix} \ color {blue} d_1 & b_1 & c_1 \\ \ color {blue} d_2 & b_2 & c_2 \\ \ color {blue} d_3 & b_3 & c_3 \ end {vmatrix} \; , \; D_y = \ begin {vmatrix} a_1 & \ color {blue} d_1 & c_1 \\ a_2 & \ color {blue} d_2 & c_2 \\ a_3 & \ color {blue} d_3 & c_3 \ end {vmatrix} \; , \; D_z = \ begin {vmatrix} a_1 & b_1 & \ color {blue} d_1 \\ a_2 & b_2 & \ color {blue} d_2 \\ a_3 & b_3 & \ color {blue} d_3 \ end {vmatrix} \]

    Если мы пишем определитель \ (D_x \), мы заменяем столбец \ (x \) постоянным столбцом.Если мы пишем определитель \ (D_y \), мы заменяем столбец y на столбец констант. Если мы пишем определитель \ (D_z \), мы заменяем столбец \ (z \) постоянным столбцом. Всегда проверяйте ответ.

    Пример \ (\ PageIndex {4} \): решение системы \ (3 × 3 \) с использованием правила Крамера

    Найдите решение данной системы \ (3 × 3 \), используя правило Крамера.

    \ [\ begin {align *} x + y-z & = 6 \\ 3x-2y + z & = -5 \\ x + 3y-2z & = 14 \ end {align *} \]

    Решение

    Используйте правило Крамера.

    \ (D = \ begin {vmatrix} 1 & 1 & −1 \\ 3 & −2 & 1 \\ 1 & 3 & −2 \ end {vmatrix} \), \ (D_x = \ begin {vmatrix} 6 & 1 & −1 \\ — 5 & −2 & 1 \ \ 14 & 3 & −2 \ end {vmatrix} \), \ (D_y = \ begin {vmatrix} 1 & 6 & −1 \\ 3 & −5 & 1 \\ 1 & 14 & −2 \ end {vmatrix} \), \ (D_z = \ begin {vmatrix } 1 & 1 & 6 \\ 3 & −2 & −5 \\ ​​1 & 3 & 14 \ end {vmatrix} \)

    Затем,

    \ [\ begin {align *} x & = \ dfrac {D_x} {D} & = \ dfrac {-3} {- 3} & = 1 \\ y & = \ dfrac {D_y} {D} & = \ dfrac {-9} {- 3} & = 3 \\ z & = \ dfrac {D_z} {D} & = \ dfrac {6} {- 3} & = -2 \\ \ end {align *} \]

    Решение: \ ((1,3, −2) \).

    Упражнение \ (\ PageIndex {3} \)

    Используйте правило Крамера, чтобы решить матрицу \ (3 × 3 \).

    \ [\ begin {align *} x-3y + 7z & = 13 \\ x + y + z & = 1 \\ x-2y + 3z & = 4 \ end {align *} \]

    Ответ

    \ (\ left (−2, \ dfrac {3} {5}, \ dfrac {12} {5} \ right) \)

    Пример \ (\ PageIndex {5A} \): использование правила Крамера для решения несовместимой системы

    Решите систему уравнений, используя правило Крамера.

    \ [\ begin {align} 3x-2y & = 4 \ label {eq3} \\ 6x-4y & = 0 \ label {eq4} \ end {align} \]

    Решение

    Начнем с нахождения определителей \ (D \), \ (D_x \) и \ (D_y \).

    \ (D = \ begin {vmatrix} 3 & −2 \\ 6 & −4 \ end {vmatrix} = 3 (−4) −6 (−2) = 0 \)

    Мы знаем, что нулевой определитель означает, что либо система не имеет решения, либо имеет бесконечное количество решений. Чтобы узнать, какой из них, мы используем процесс исключения. Наша цель — исключить одну из переменных.

    1. Умножьте уравнение \ ref {eq3} на \ (- 2 \).
    2. Добавьте результат в уравнение \ ref {eq4}.

    \ [\ begin {align *} & −6x + 4y = −8 \\ & \; \; \; \ underline {6x − 4y = 0} \\ & \; \; \; \; \; \ ; \; \; \; \; 0 = −8 \ end {align *} \]

    Получаем уравнение \ (0 = −8 \), которое неверно. Следовательно, у системы нет решения. График системы показывает две параллельные линии. См. Рисунок \ (\ PageIndex {1} \).

    Рисунок \ (\ PageIndex {1} \)

    Пример \ (\ PageIndex {5B} \): использование правила Крамера для решения зависимой системы

    Решите систему с бесконечным количеством решений.

    \ [\ begin {align} x-2y + 3z & = 0 \ label {eq5} \\ 3x + y-2z & = 0 \ label {eq6} \\ 2x-4y + 6z & = 0 \ label {eq7} \ end {align} \]

    Решение

    Давайте сначала найдем определитель. Создайте матрицу, дополненную первыми двумя столбцами.

    \ (\ left | \ begin {array} {ccc | cc} 1 & −2 & 3 & 1 & -2 \\ 3 & 1 & −2 & 3 & 1 \\ 2 & −4 & 6 & 2 & -4 \ end {array} \ right | \)

    Затем,

    \ (1 (1) (6) + (- 2) (- 2) (2) +3 (3) (- 4) −2 (1) (3) — (- 4) (- 2) (1 ) −6 (3) (- 2) = 0 \)

    Поскольку определитель равен нулю, решения либо нет, либо существует бесконечное количество решений.Чтобы выяснить это, нам нужно провести отбор.

    1. Умножьте уравнение \ ref {eq5} на \ (- 2 \) и добавьте результат к уравнению \ ref {eq7}:

    \ [\ begin {align *} & −2x + 4y − 6x = 0 \\ & \; \; \ underline {2x − 4y + 6z = 0} \\ & \; \; \; \; \; \ ; \; \; \; \; \; \; \; \; \; \; 0 = 0 \ end {align *} \]

    2. Получение ответа \ (0 = 0 \), утверждение, которое всегда верно, означает, что система имеет бесконечное количество решений. Изобразив систему, мы можем увидеть, что две плоскости одинаковы, и обе они пересекают третью плоскость по прямой.См. Рисунок \ (\ PageIndex {2} \).

    Рисунок \ (\ PageIndex {2} \)

    Понимание свойств детерминантов

    Есть много свойств определителей. Здесь перечислены некоторые свойства, которые могут быть полезны при вычислении определителя матрицы.

    СВОЙСТВА ДЕТЕРМИНАНТОВ

    1. Если матрица имеет верхнюю треугольную форму, определитель равен произведению входов по главной диагонали.
    2. Когда две строки меняются местами, определитель меняет знак.{−1} \) — величина, обратная определителю матрицы \ (A \).
    3. Если какая-либо строка или столбец умножается на константу, определитель умножается на тот же коэффициент.

    Пример \ (\ PageIndex {6} \): иллюстрация свойств детерминантов

    Проиллюстрируйте каждое из свойств определителей.

    Решение

    Свойство 1 утверждает, что если матрица имеет верхнюю треугольную форму, определитель является произведением элементов по главной диагонали.

    \ (A = \ begin {bmatrix} 1 & 2 & 3 \\ 0 & 2 & 1 \\ 0 & 0 & −1 \ end {bmatrix} \)

    Дополните \ (A \) первыми двумя столбцами.

    \ (A = \ left [\ begin {array} {ccc | cc} 1 & 2 & 3 & 1 & 2 \\ 0 & 2 & 1 & 0 & 2 \\ 0 & 0 & −1 & 0 & 0 \ end {array} \ right] \)

    Затем

    \ [\ begin {align *} \ det (A) & = 1 (2) (- 1) +2 (1) (0) +3 (0) (0) -0 (2) (3) -0 (1) (1) +1 (0) (2) \\ & = -2 \ end {align *} \]

    Свойство 2 утверждает, что перестановка строк меняет знак.Учитывая

    \ [\ begin {align *} B & = \ begin {bmatrix} 4 & -3 \\ — 1 & 5 \ end {bmatrix} \\ \ det (B) & = (4) (5) — (- 1) (- 3) \\ & = 20-3 \\ & = 17 \ end {align *} \]

    Свойство 3 утверждает, что если две строки или два столбца идентичны, определитель равен нулю.

    \ [\ begin {align *} A & = \ left [\ begin {array} {ccc | cc} 1 & 2 & 2 & 1 & 2 \\ 2 & 2 & 2 & 2 & 2 \\ — 1 & 2 & 2 & -1 & 2 \ end {array} \ right] \\ \ det (A) & = 1 (2) (2) +2 (2) (- 1) +2 (2) (2) +1 (2) (2) -2 (2) (1) -2 (2) (2) \ \ & = 4-4 + 8 + 4-4-8 \\ & = 0 \ end {align *} \]

    Свойство 4 утверждает, что если строка или столбец равны нулю, определитель равен нулю.{-1}) & = — 2 \ left (- \ dfrac {1} {2} \ right) — \ dfrac {3} {2} (1) \\ & = — \ dfrac {1} {2} \ конец {выравнивание *} \]

    Свойство 6 утверждает, что если любая строка или столбец матрицы умножается на константу, определитель умножается на тот же коэффициент. Таким образом,

    Пример \ (\ PageIndex {7} \): использование правила Крамера и определяющих свойств для решения системы

    Найдите решение данной системы \ (3 × 3 \).

    Решение

    Используя правило Крамера, имеем

    \ (D = \ begin {bmatrix} 2 & 4 & 4 \\ 3 & 7 & 7 \\ 1 & 2 & 2 \ end {bmatrix} \)

    Обратите внимание, что второй и третий столбцы идентичны.Согласно свойству 3 определитель будет равен нулю, поэтому решения либо нет, либо существует бесконечное число решений. Чтобы выяснить это, нам нужно провести отбор.

    1. Умножьте уравнение \ ref {eq10} на \ (- 2 \) и добавьте результат в уравнение \ ref {eq8}.

    Получение противоречивого утверждения означает, что система не имеет решения.

    Медиа

    Получите доступ к этим онлайн-ресурсам для получения дополнительных инструкций и практики с правилом Крамера.

    Решение систем уравнений (одновременных уравнений)

    Если у вас есть два разных уравнения с одинаковыми двумя неизвестными в каждом, вы можете решить для обоих неизвестных.Существует три распространенных метода решения: сложение / вычитание, подстановка и построение графика.

    Метод сложения / вычитания

    Этот метод также известен как метод исключения.

    Чтобы использовать метод сложения / вычитания, выполните следующие действия:

    1. Умножьте одно или оба уравнения на некоторое число (а), чтобы число перед одной из букв (неизвестных) в каждом уравнении было одинаковым или прямо противоположным.

    2. Сложите или вычтите два уравнения, чтобы исключить одну букву.

    3. Решите оставшееся неизвестное.

    4. Решите для другого неизвестного, вставив значение неизвестного, найденного в одно из исходных уравнений.

    Пример 1

    Решите для x и y .

    При добавлении уравнений исключаются термины и .

    Теперь добавление 5 для x в первое уравнение дает следующее:

    Ответ: x = 5, y = 2

    Заменяя каждое x на 5 и каждое y на 2 в исходных уравнениях, вы можете увидеть, что каждое уравнение станет истинным.

    В примере и пример , существовал уникальный ответ для x и y , который делал каждое предложение одновременно истинным. В некоторых ситуациях вы не получаете однозначных ответов или вообще не получаете ответов. Вы должны знать об этом, когда используете метод сложения / вычитания.

    Пример 2

    Решите для x и y.

    Сначала умножьте нижнее уравнение на 3. Теперь перед y стоит 3 в каждом уравнении.

    Уравнения можно вычесть, исключив члены y .

    Вставьте x = 5 в одно из исходных уравнений, чтобы найти y .

    Ответ: x = 5, y = 3

    Конечно, если число перед буквой уже одно и то же в каждом уравнении, вам не нужно изменять ни одно уравнение. Просто сложите или вычтите.

    Чтобы проверить решение, замените каждое x в каждом уравнении на 5 и замените каждое y в каждом уравнении на 3.

    Пример 3

    Решите относительно a и b .

    Умножьте верхнее уравнение на 2. Обратите внимание на то, что происходит.

    Теперь, если вы вычтете одно уравнение из другого, результат будет 0 = 0.

    Это утверждение всегда верно .

    Когда это происходит, система уравнений не имеет единственного решения. Фактически, любая замена a и b , которая делает одно из уравнений истинным, также делает истинным другое уравнение.Например, если a = –6 и b = 5, то оба уравнения выполняются.

    [3 (- 6) + 4 (5) = 2 И 6 (- 6) + 8 (5) = 4]

    В действительности мы имеем только одно уравнение, записанное двумя разными способами. В этом случае второе уравнение фактически является первым уравнением, умноженным на 2. Решением этой ситуации является либо исходное уравнение, либо упрощенная форма любого уравнения.

    Пример 4

    Решите для x и y .

    Умножьте верхнее уравнение на 2. Обратите внимание на то, что происходит.

    Теперь, если вы вычтете нижнее уравнение из верхнего уравнения, результат будет 0 = 1. Это утверждение никогда не соответствует действительности . Когда это происходит, система уравнений не имеет решения.

    В примерах 1–4 только одно уравнение было умножено на число, чтобы числа перед буквой были одинаковыми или противоположными. Иногда каждое уравнение необходимо умножить на разные числа, чтобы числа перед буквой были одинаковыми или противоположными.

    Решите для x и y .

    Обратите внимание, что не существует простого числа, на которое можно умножить любое уравнение, чтобы получить числа перед x или y , чтобы они стали одинаковыми или противоположными. В этом случае сделайте следующее:

    1. Выберите букву, которую нужно удалить.

    2. Используйте две цифры слева от этой буквы. Найдите наименьшее общее кратное этого значения как желаемое число перед каждой буквой.

    3. Определите, на какое значение необходимо умножить каждое уравнение, чтобы получить это значение, и умножьте уравнение на это число.

    Предположим, вы хотите удалить x . Наименьшее общее кратное 3 и 5, число перед x , равно 15. Первое уравнение нужно умножить на 5, чтобы получить 15 перед x . Второе уравнение нужно умножить на 3, чтобы получить 15 перед x .

    Теперь вычтите второе уравнение из первого, чтобы получить следующее:

    На этом этапе вы можете либо заменить y на и решить для x (метод 1, который следует ниже), либо начать с двух исходных уравнений и исключить y , чтобы решить для x (метод 2, который следует).

    Метод 1

    Используя верхнее уравнение: замените y на и решите относительно x .

    Метод 2

    Исключаем y и решаем относительно x .

    Наименьшее общее кратное 4 и 6 равно 12. Умножьте верхнее уравнение на 3, а нижнее уравнение на 2.

    Теперь сложите два уравнения, чтобы исключить и .

    Решение: x = 1 и.

    Метод замещения

    Иногда систему проще решить с помощью метода подстановки . Этот метод включает замену одного уравнения в другое.

    Пример 6

    Решите для x и y.

    Из первого уравнения замените ( y + 8) на x во втором уравнении.

    ( y + 8) + 3 y = 48

    Теперь решите г. Упростите, объединив и .

    Теперь вставьте значение y , 10, в одно из исходных уравнений.

    Ответ: y = 10, x = 18

    Проверьте решение.

    Пример 7

    Решите для x и y , используя метод подстановки.

    Сначала найдите уравнение, в котором перед буквой стоит цифра «1» или «- 1». Решите эту букву с точки зрения другой буквы.

    Затем действуйте как в примере 6.

    В этом примере в нижнем уравнении перед числами y стоит «1».

    Решите относительно y в терминах x .

    Замените 4 x — 17 на y в верхнем уравнении, а затем решите относительно x .

    Замените x на 4 в уравнении y — 4 x = –17 и решите относительно y .

    Решение: x = 4, y = –1.

    Проверьте решение:

    Метод построения графика

    Другой метод решения уравнений — это построения каждого уравнения на координатном графике. Координаты перекрестка будут решением системы. Если вы не знакомы с построением координатных графиков, внимательно просмотрите статьи по координатной геометрии, прежде чем пытаться использовать этот метод.

    Пример 8

    Решите систему, построив график.

    Сначала найдите три значения для x и y , которые удовлетворяют каждому уравнению. (Хотя для определения прямой необходимы только две точки, поиск третьей точки — хороший способ проверки.) Ниже приведены таблицы значений x и y :

    Теперь изобразите две линии на координатной плоскости, как показано на рисунке 1.

    Точка пересечения двух линий (4, 0) — это решение системы.

    Если линии параллельны, они не пересекаются, и, следовательно, для этой системы нет решения.

    Рис. 1. График из линий x = 4 + y и x — 3 y = 4, обозначающих решение.

    Пример 9

    Решите систему, построив график.

    Найдите три значения для x и y , которые удовлетворяют каждому уравнению.

    3 x + 4 y = 2 6 x + 8 y = 4

    Ниже приведены таблицы значений x и y .См. Рисунок 2.

    х

    л

    0

    2

    — 1

    4

    х

    л

    0

    2

    — 1

    4

    Обратите внимание, что одинаковые точки удовлетворяют каждому уравнению.Эти уравнения представляют собой одну и ту же линию.

    Следовательно, решение не единственное. Решение — это все точки на линии.

    Следовательно, решением является любое уравнение прямой, поскольку они оба представляют одну и ту же линию.

    Это похоже на пример когда это было сделано с использованием метода сложения / вычитания.

    Рисунок 2. График из линий 3 x + 4 y = 2 и 6 x + 8 y = 4, указывающих решение.

    Пример 10

    Решите систему, построив график.

    Найдите три значения для x и y , которые удовлетворяют каждому уравнению. См. Следующие таблицы значений x и y :

    х

    л

    0

    1

    2

    4

    -2

    х

    л

    0

    2

    2

    4

    –1

    Обратите внимание на то, что на рисунке 3 два графика параллельны.Они никогда не встретятся. Следовательно, у этой системы уравнений нет решения.

    Для этой системы уравнений не существует решения.

    Это похоже на пример выполняется методом сложения / вычитания.

    Рисунок 3. График из линий 3 x + 4 y = 4 и 6 x + 8 y = 16, указывающих решение.


    Промежуточная алгебра
    Урок 7: Линейные уравнения в одной переменной

    WTAMU > Виртуальная математическая лаборатория> Алгебра среднего уровня

    Цели обучения


    После изучения этого руководства вы сможете:
    1. Знайте, что такое линейное уравнение.
    2. Знайте, является ли значение решением или нет.
    3. Используйте свойства сложения, вычитания, умножения и деления равенств для решения линейных уравнений.
    4. Знайте, когда уравнение не имеет решения.
    5. Знайте, когда уравнение имеет все действительные числа в качестве решения.

    Введение



    Здесь мы начинаем вникать в суть того, что алгебра о — решение уравнений.В этом уроке мы будем искать конкретно при линейных уравнениях и их решениях. Мы начнем медленно а также решать уравнения, использующие только одно свойство, чтобы убедиться, что у вас есть физическое лицо понятий вниз. Затем мы наберем темп и смешаем их там, где вам нужно использовать несколько свойств и шагов, чтобы выполнить работу.

    Уравнения могут быть использованы для решения различных проблемы. Позже учебные пособия, мы будем использовать их для решения текстовых задач.потом ты может ответить на эти сложные математические вопросы.

    Учебник





    Уравнение

    Два выражения равны друг другу


    Линейное уравнение

    Уравнение, которое можно записать в виде
    ax + b = c
    , где a, b и c — константы



    Ниже приведен пример линейного уравнения: 3 x — 4 = 5




    Решение

    Значение, такое, что при замене переменной на it,
    это делает уравнение верно.

    (левая сторона выходит равной правой)


    Набор решений

    Набор всех решений



    Пример 1 : Определите, соответствует ли какое-либо из следующих значений x решения к данному уравнению.
    3 x — 4 знак равно 5; x = 3, 5.

    Проверка 3
    3 x — 4 = 5
    3 (3) — 4 = 5
    9–4 = 5
    5 = 5
    Истинно 3 это решение

    Проверка 5
    3 x — 4 = 5
    3 (5) — 4 = 5
    15–4 = 5
    11 = 5
    Неверно 5 не решение




    Решение линейного уравнения
    в целом

    Получите переменную, которую вы решаете, в одиночку с одной стороны и все else на другой стороне, используя ОБРАТНЫЕ операции.


    Ниже приведены инструменты, необходимые для решать линейные уравнения.


    Сложение и вычитание Свойства равенства

    Если a = b, то a + c = b + c

    Если a = b, то a — c = b — c


    Другими словами, если два выражения равны каждому другой и ты прибавлять или вычитать одно и то же к обеим сторонам, обе стороны будут оставаться равными.

    Обратите внимание, что сложение и вычитание являются обратными операции каждого Другие. Например, если у вас есть добавляемый номер, вам нужно перейти к другой стороне уравнения, тогда вы бы вычесть это с обеих сторон этого уравнения.

    Пример 2 : Найдите переменную. x — 5 = 2.


    x — 5 = 2
    x — 5 + 5 = 2 + 5
    x = 7

    * Обратное от sub. 5 — доп. 5


    Обратите внимание, что если вы вернете 7 для x дюймов исходной проблемы вы увидите, что 7 — это решение нашей проблема.




    Пример 3 : Найдите переменную. y + 4 = -7.

    y + 4 = -7
    y + 4-4 = -7-4
    y = -11

    * Инверсия доп.4 является суб. 4


    Обратите внимание, что если вы вернете -11 для y в исходной задаче, вы увидите, что -11 — это решение, которое мы находятся ищу .




    Умножение и деление Свойства равенства

    Если a = b, то a (c) = b (c)

    Если a = b, то a / c = b / c, где c — не равно 0.


    Другими словами, , если два выражения равны друг друга и ты умножить или разделить (кроме 0) одну и ту же константу на оба стороны, обе стороны останутся равными.

    Обратите внимание, что умножение и деление являются обратными операции каждого Другие.Например, если у вас есть число, которое умножается что вам нужно перейти к другой стороне уравнения, тогда вы бы разделите его с обеих сторон этого уравнения.

    Обратите внимание, что для умножения и деления это не гарантировал, что если вы умножаете на переменную, которую вы решаете, чтобы две стороны будет равным. Но гарантировано, что обе стороны пойдут быть равным, если вы умножаете или делите на константу или другое переменная, для которой вы не решаете.Мы поговорим подробнее о это в более позднем руководстве. Для этого урока просто обратите внимание, что вы можете использовать это свойство с константами и переменными, для которых вы не ищите.

    Пример 4 : Найдите переменную. х /2 = 5.


    * Обратно дел.на 2 это мульт. по 2


    Если вы вернете 10 для x дюймов оригинал проблема, вы увидите, что 10 — это решение, которое мы ищем.



    Пример 5 : Найдите переменную.5 x = 7.

    * Инверсная по отношению к мульт. на 5 дел. по 5


    Если вы вставите 7/5 обратно для x в оригинале проблема, вы увидите, что 7/5 — это решение, которое мы ищем.



    В приведенных выше примерах использовались только одно свойство за раз, чтобы помочь вам понять различные свойства, которые мы используем к решать уравнения.Однако в большинстве случаев нам приходится использовать несколько характеристики чтобы выполнить свою работу. Ниже приводится стратегия, которую вы можете использовать. чтобы помочь вам решить более сложные линейные уравнения.



    Стратегия решения линейной Уравнение

    Обратите внимание, что ваш учитель или книга ты использование, возможно, сформулировало эти шаги немного иначе, чем я, но Это все сводится к одной и той же концепции — включите свою переменную один сторона и все остальное с другой, используя обратные операции.

    Шаг 1. При необходимости упростите каждую сторону.

    Это может включать в себя такие вещи, как удаление (), удаление дробей, добавление как термины и т. д.

    Чтобы удалить (): Просто используйте дистрибутив свойство, найденное в Уроке 5: Свойства действительных чисел.

    Для удаления дробей : Поскольку дроби другой способ написать деление, а обратное деление — умножение, вы удаляете фракции умножив обе части на ЖК-дисплей всех ваших дробей.

    Шаг 2: Используйте Добавить./ Sub. Свойства для переместить переменную срок в одну сторону и все остальные условия в другую сторону.

    Шаг 3: Используйте Mult./Div. Свойства для удалить любые значения которые находятся перед переменной.

    Шаг 4. Проверьте свой ответ.

    Я считаю, что это самый быстрый и Самый простой способ приблизиться к линейным уравнениям.


    Пример 6 : Найдите переменную. 10 — 3 x = 7.


    * Инверсия доп. 10 является суб. 10

    * Обратное от мульт.на -3 — это div. по -3


    Будьте осторожны, начиная со строки 4 к строке 5. Да, есть отрицательный знак. Но операция между -3 и x — это умножение, а не вычитание. Итак, если бы вы Добавлять 3 в обе стороны, вы бы получили -3 x + 3 вместо желаемых x .

    Если вы вернете 1 вместо x в исходной задаче, вы увидим, что 1 это решение, которое мы ищем.




    Пример 7 : Найдите переменную. 2 ( x + 5) — 7 = 3 ( x — 2).

    * Удалить () с помощью dist.опора

    * Получить все условия x с одной стороны

    * Инверсия доп. 3 является суб. 3

    * Обратное от мульт. на -1 — это div. по -1


    Если вы вернете 9 вместо x в исходной задаче, вы увидим, что 9 — это решение, которое мы ищем.



    Пример 8 : Найдите переменную:.

    * Чтобы избавиться от дроби,
    мульт. с обеих сторон ЖК-дисплеем 4

    * Получить все термины x на одной стороне

    * Инверсия доп.2 является суб. 2

    * Обратное от мульт. на -3 — это div. по -3


    Если вы вернете 4/3 вместо x в исходной задаче вы увидите, что 4/3 это решение, которое мы ищем.



    Противоречие

    Противоречие — это уравнение с одной переменной, которая не имеет решения.



    Пример 9 : Найдите переменную. 4 x — 1 = 4 ( x + 3).


    * Удалить () с помощью dist. опора

    * Получить все термины x на одной стороне


    Куда делась наша переменная x, ??? Он исчез на нас.Также обратите внимание, как мы получили ЛОЖЬ утверждение, -1 не равно 12. Это не означает, что x = 12 или x = -1.

    Когда ваша переменная падает из И вы закончите с ложным утверждением, то после всей вашей тяжелой работы есть НЕТ РЕШЕНИЕ.

    Итак, ответ — нет решения.




    Личность

    Тождество — это уравнение с одной переменной
    который имеет все действительные числа как решение.



    Пример 10 : Найдите переменную. 5 x + 10 = 5 ( x + 2).


    * Удалить () с помощью dist. опора

    * Получить все термины x на одной стороне


    На этот раз, когда наша переменная выпал, мы закончил с ИСТИННЫМ заявлением.Когда бы это ни случилось, твой ответ ВСЕ РЕАЛЬНЫЕ ЧИСЛА.

    Итак, ответ — все действительные числа .



    Практические задачи


    Это практические задачи, которые помогут вам следующий уровень. Это позволит вам проверить и понять, понимаете ли вы эти типы проблем. Math работает так же, как что-нибудь иначе, если вы хотите добиться успеха в этом, вам нужно практиковаться Это. Даже лучшие спортсмены и музыканты получали помощь и много практиковаться, практиковаться, практиковаться, чтобы стать лучше в своем виде спорта или инструменте. На самом деле не бывает слишком много практики.

    Чтобы получить от них максимальную отдачу, вам следует проблема на свой собственный, а затем проверьте свой ответ, щелкнув ссылку для ответ / обсуждение для этой проблемы .По ссылке вы найдете ответ а также любые шаги, которые позволили найти этот ответ.

    Практика Задачи 1a — 1e: Решите для переменной.


    Нужна дополнительная помощь по этим темам?





    Последний раз редактировал Ким Сьюард 1 июля 2011 г.
    Авторские права на все содержимое (C) 2002 — 2011, WTAMU и Kim Seward. Все права защищены.

    Алгебра — линейные уравнения

    Решите каждое из следующих уравнений.

    Показать обсуждение

    В следующих задачах мы подробно опишем первую проблему и оставим большую часть объяснений нижеприведенных проблем.


    a \ (3 \ left ({x + 5} \ right) = 2 \ left ({- 6 — x} \ right) — 2x \) Показать решение

    Для этой задачи нет дробей, поэтому нам не нужно беспокоиться о первом этапе процесса.На следующем шаге нужно упростить обе стороны. Итак, мы уберем все скобки, умножив числа, а затем объединим похожие термины.

    \ [\ begin {align *} 3 \ left ({x + 5} \ right) & = 2 \ left ({- 6 — x} \ right) — 2x \\ 3x + 15 & = — 12 — 2x — 2x \\ 3x + 15 & = — 12 — 4x \ end {align *} \]

    Следующий шаг — получить все \ (x \) с одной стороны и все числа с другой стороны. С какой стороны идти \ (x \) — решать вам и, вероятно, будет зависеть от проблемы.Как правило, мы помещаем переменные в ту сторону, которая дает положительный коэффициент. Это делается просто потому, что часто легко потерять знак минус на коэффициенте, и поэтому, если мы убедимся, что он положительный, нам не нужно об этом беспокоиться.

    Итак, для нашего случая это будет означать прибавление 4 \ (x \) к обеим сторонам и вычитание 15 с обеих сторон. Также обратите внимание, что, хотя мы фактически выполняем эти операции в это время, мы обычно выполняем эти операции в нашей голове.

    \ [\ begin {align *} \ require {color} 3x + 15 & = — 12 — 4x \\ 3x + 15 {\ color {Red} — 15} {\ color {Blue} + 4x} & = — 12 — 4x {\ color {Blue} + 4x} {\ color {Red} — 15} \\ 7x & = — 27 \ end {align *} \]

    На следующем этапе нужно получить коэффициент 1 перед \ (x \). В этом случае мы можем сделать это, разделив обе стороны на 7.

    \ [\ begin {align *} \ frac {{7x}} {7} & = \ frac {{- 27}} {7} \\ x & = — \ frac {{27}} {7} \ end { выровнять*}\]

    Теперь, если мы выполнили всю нашу работу правильно, \ (x = — \ frac {{27}} {7} \) является решением уравнения.

    Последний и последний шаг — проверить решение. Как указано в схеме процесса, нам нужно проверить решение в исходном уравнении . Это важно, потому что мы могли допустить ошибку на самом первом шаге, и если мы сделали, а затем проверили ответ в результатах этого шага, может показаться, что решение верное, хотя на самом деле мы этого не делаем. У меня нет правильного ответа из-за ошибки, которую мы сделали изначально.? 2 \ left ({- \ frac {{15}} {7}} \ right) + \ frac {{54}} {7} \\ \ frac {{24}} {7} & = \ frac {{24 }} {7} \ hspace {0.5in} {\ mbox {OK}} \ end {align *} \]

    Итак, мы сделали свою работу правильно и решение уравнения:

    \ [x = — \ frac {{27}} {7} \]

    Обратите внимание, что здесь мы не использовали обозначение набора решений. Для отдельных решений мы редко будем делать это в этом классе. Однако, если бы мы хотели, чтобы обозначение набора решений для этой задачи было бы

    \ [\ left \ {{- \ frac {{27}} {7}} \ right \} \]

    Прежде чем перейти к следующей задаче, давайте сначала кратко прокомментируем «беспорядок» этого ответа.НЕ ожидайте, что все ответы будут красивыми простыми целыми числами. Хотя мы стараемся, чтобы большинство ответов были простыми, часто это не так, поэтому НЕ зацикливайтесь на идее, что ответ должен быть простым целым числом, что вы сразу же предполагаете, что вы сделали ошибку из-за «беспорядка» ответ.


    b \ (\ displaystyle \ frac {{m — 2}} {3} + 1 = \ frac {{2m}} {7} \) Показать решение

    Хорошо, с этим мы не будем так подробно объяснять проблему.

    В этом случае у нас есть дроби, поэтому, чтобы облегчить нашу жизнь, мы умножим обе части на ЖК-дисплей, который в данном случае равен 21. После этого проблема будет очень похожа на предыдущую. Также обратите внимание, что знаменатели — это только числа, поэтому нам не нужно беспокоиться о делении на ноль.

    Давайте сначала умножим обе стороны на ЖК-дисплей.

    \ [\ begin {align *} 21 \ left ({\ frac {{m — 2}} {3} + 1} \ right) & = \ left ({\ frac {{2m}} {7}} \ right ) 21 \\ 21 \ left ({\ frac {{m — 2}} {3}} \ right) + 21 \ left (1 \ right) & = \ left ({\ frac {{2m}} {7} } \ right) 21 \\ 7 \ left ({m — 2} \ right) + 21 & = \ left ({2m} \ right) \ left (3 \ right) \ end {align *} \]

    Будьте осторожны, чтобы правильно распределить 21 в скобках с левой стороны.Все, что находится внутри скобок, нужно умножить на 21, прежде чем мы упростим. На данный момент у нас есть проблема, аналогичная предыдущей, и на этот раз мы не будем утруждать себя ее объяснениями.

    \ [\ begin {align *} 7 \ ​​left ({m — 2} \ right) + 21 & = \ left ({2m} \ right) \ left (3 \ right) \\ 7m — 14 + 21 & = 6m \\ 7m + 7 & = 6m \\ m & = — 7 \ end {align *} \]

    Итак, похоже, \ (m = — 7 \) — это решение.2} — 6 \ left (5 \ right) + 9}} \\ \ frac {5} {4} & = \ frac {5} {4} \ hspace {0.5in} {\ mbox {OK}} \ end {выровнять*}\]
    d \ (\ displaystyle \ frac {{2z}} {{z + 3}} = \ frac {3} {{z — 10}} + 2 \) Показать решение

    В этом случае ЖК-дисплей выглядит как \ (\ left ({z + 3} \ right) \ left ({z — 10} \ right) \), и также похоже, что нам нужно избегать \ (z = — 3 \) и \ (z = 10 \), чтобы не получить деление на ноль.

    Приступим к работе над этой проблемой.2} — 7z — 30} \ right) \ end {align *} \]

    На этом этапе давайте сделаем паузу и признаем, что здесь в работе находится z 2 . 2}}} — 14z — 60 \ \ — 20z & = — 11z — 51 \\ 51 & = 9z \\ \ frac {{51}} {9} & = z \\ & \ frac {{17}} {3} = z \ end {align * } \]

    Обратите внимание, что z 2 действительно аннулировал.? 3 \ left ({- \ frac {3} {{13}}} \ right) + 2 \\ \ frac {{17}} {{13}} & = \ frac {{17}} {{13}} \ hspace {0,5 дюйма} {\ mbox {OK}} \ end {align *} \]

    Иногда проверка может быть немного запутанной, но это означает, что мы ЗНАЕМ, что решение правильное.

    .

Добавить комментарий

Ваш адрес email не будет опубликован. Обязательные поля помечены *